You are on page 1of 46

SA IAS ACADEMY

Your Dreams Our Mission

Prelims
Test Series
GENERAL Current affairs TEST

STUDIES
Topics: -
January current affairs 2024
Feburary current affairs 2024

WWW.SAIASACADEMY.COM
SA IAS ACADEMY UPSC (15/03/2024)

GENERAL Current affairs


test
STUDIES

Time Allowed: Two Hours Maximum Marks: 200

INSTRUCTION
1. IMMEDIATELY AFTER THE COMMENCEMENT OF THE EXAMINATION, YOU SHOULD
CHECK THAT THIS TEST BOOKLET DOES NOT HAVE ANY UNPRINTED OR TORN OR
MISSING PAGES OR ITEMS, ETC. IF SO, GET IT REPLACED BY A COMPLETE TEST BOOKLET.
2. Please note that it is the candidate’s responsibility to encode and fill in the Roll Number carefully
without any omission or discrepancy at the appropriate places in the OMR Answer Sheet. Any
omission/discrepancy will render the Answer Sheet liable for rejection.
3. You have to enter your Roll Number on the test booklet
in the Box provided alongside. DO NOT writes anything
else on the Test Booklet.
4. This Test Booklet contains 100 items (questions). Each item is printed in English. Each item comprises
four responses (answers). You will select the response which you want to mark on the Answer Sheet. In
case you feel that there is more than one correct response, mark the response which you consider the best.
In any case, choose ONLY ONE response for each item
5. You have to mark all your responses ONLY on the separate Answer Sheet Provided. See directions in the
Answer Sheet.
6. All items carry equal marks.
7. Before you proceed to mark in the Answer Sheet in response to various items in the Test Booklet, you have
to fill in some particulars in the Answer Sheet as per instructions sent to you with your Admission
Certificate.
8. After you have completed filling in all your responses on the Answer Sheet and the examination has
concluded, you should hand over to the Invigilator only the Answer Sheet. You are permitted to take away
with you the Test Booklet
9. Sheets for rough work are appended in the Test Booklet at the end.
10. Penalty for wrong answers:
THERE WILL BE PENALTY FOR WRONG ANSWERS MARKED BY A CANDIDATE IN THE
OBJECTIVE TYPE QUESTION PAPERS.
(i) There are four alternatives for the answer to every question. For each question for which a
wrong answer has been given by the candidate, one-third of the marks assigned to that question
will be deducted as penalty.
(ii) If a candidate gives more than one answer, it will be treated as a wrong answer even if one of
the given answers happens to be correct and there will be same penalty as above to that
question.
(iii) If a question is left blank, i.e., no answer is given by the candidate, there will be no penalty for
that question.

DO NOT OPEN THIS TEST BOOKLET UNTIL YOU ARE ASKED TO DO SO


For More Details Contact Us On: 0422-4954645; 9958604645
SA IAS ACADEMY UPSC (15/03/2024)

ANSWER KEY

1. C 11. C 21. C 31. A 41. C 51. A 61. A 71. A 81. C 91. C

2. D 12. A 22. A 32. B 42. B 52. B 62. A 72. C 82. B 92. B

3. B 13. B 23. A 33. C 43. B 53. B 63. B 73. B 83. B 93. B

4. B 14. D 24. B 34. A 44. A 54. B 64. B 74. C 84. A 94. A

5. C 15. C 25. C 35. B 45. B 55. B 65. B 75. B 85. B 95. C

6. C 16. A 26. A 36. B 46. C 56. B 66. C 76. B 86. B 96. B

7. B 17. B 27 C 37. B 47. A 57. B 67. A 77. C 87. C 97. A

8. A 18. C 28. C 38. C 48. C 58. B 68. B 78. A 88. C 98. A

9. C 19. C 29. B 39. C 49. B 59. C 69. B 79. B 89. A 99. B

10. B 20. B 30. B 40. A 50. A 60. C 70. A 80. A 90. A 100. B

For More Details Contact Us On: 0422-4954645; 9958604645


SA IAS ACADEMY UPSC (15/03/2024)
1. With reference to Bubble Baby Syndrome, consider the following statements:
1. It is a rare genetic disorder.
2. It affects infection-fighting white blood cells in the body.
3. The most common treatment for his condition is a stem cell transplant.
How many of the above statements are correct?
(a) Only one
(b) Only two
(c) All three
(d) None of the above.
Answer : C
Explanation:
A two-month-old girl with bubble baby syndrome underwent bone marrow transplant (BMT) in Mumbai recently.
About Bubble Baby Syndrome:
'Bubble baby syndrome, known medically as Severe Combined Immunodeficiency (SCID) is very rare genetic disorder that
causes life-threatening problems with the immune system.
What happens in SCID?
• In a developing baby, the immune system starts in the bone marrow. Stem cells can become any of three different
types of blood cells.
• red blood cells
• white blood cells
. platelets
• White blood cells (WBCs) protect the body from infections and foreign invaders. There are different kinds of WBCs,
including lymphocytes.
• Lymphocytes come in two main types: B-cells and T-cells. These cells are key to fighting infections.
• T cells identify, attack, and kill "invaders."
• B cells make antibodies that "remember" an infection and are ready in case the body is exposed to it again.
• SCID is a "combined" immunodeficiency because it affects both of these infection-fighting white blood cells.
• The most common treatment is a stem cell transplant (also called a bone marrow transplant). This means the child
receives stem cells from a donor. The hope is that these new cells will rebuild the child's immune system.

2. The KH-22 supersonic cruise missile is developed by:


(a)Iran
(b) Israel
(c) Ukraine
(d) Russia
Answer : D
Explanation:
About KH-22 Missile:
It is a Soviet-era long-range airborne supersonic cruise missile.
• Armed with a nuclear or highly explosive fragmenting cumulative warhead, it was initially designed to destroy aircraft
carriers and other large warships, or even groups of such carriers.
• The Kh-22 family was developed in the USSR in the 1960s and specifically designed to be launched from Tupolev-22
bombers.
• Later, both the missiles and the aircraft were modernized as part of the so-called "Kh-22 special air-to-surface missile
complex.

3. Chandaka-Dampara Wildlife Sanctuary is located in:


(a) Himachal Pradesh
(b)Odisha

For More Details Contact Us On: 0422-4954645; 9958604645


SA IAS ACADEMY UPSC (15/03/2024)
(c)Assam
(d)Meghalaya
Answer : B
Explanation:
As a first, the Forest Department in Odisha has planned a 'nocturnal trail' for people, especially students and researchers,
in the Chandaka-Dampara Wildlife Sanctuary.
About Chandaka-Dampara Wildlife Sanctuary:
• Location: It lies partly within Khurda and partly in Cuttack Districts of Odisha State, and is in close proximity to the state
capital, Bhubaneswar.

4. With reference to the River Devika Project, consider the following statements:
1. It is a river rejuvenation project implemented in Jammu and Kashmir.
2. It plans to develop bathing "ghats" and catchment areas.
3. Its entire cost is borne by the State Government.
How many of the above statements are correct?
(a)Only one
(b) Only two
(c)All three
(d)None
Answer B
Explanation:
North India's first river rejuvenation project, 'Devika', is in the final stages. and it will be soon dedicated to the nation by
the Prime Minister.
About River Devika Project:
Built on the lines of 'Namami Ganga', the project was launched in February 2019.
It is north India's first river rejuvenation project.
• The project is implemented along the Devika River in Jammu and Kashmir.
It has been included in the Government of India's National River Conservation Project (NRCP).

5. What is the maximum punishment under the Bharatiya Nyay Sanhita (BNS) for a driver who causes a serious road
accident due to careless driving and leaves without informing the police or any official?
(a) 1 years in jail and a fine of Rs 5 lakh
(b) 5 years in jail and a fine of Rs 10 lakh
(c) 10 years in jail and a fine of Rs 7 lakh
(d) 20 years in jail and a fine of Rs 2 lakh
Answer : C
Explanation:
The countrywide truckers' stir has been called off recently as the government assured that it would consult stakeholders
before implementing a contentious law against hit-and-run.
About the new hit-and-run law:
• As per the Bharatiya Nyay Sanhita (BNS), which is a replacement for the British-era Indian Penal Code (IPC), if a driver
causes a serious road accident due to careless driving and then leaves without informing the police or any official, they
could be punished with up to 10 years in jail and a fine of Rs 7 lakh.
• BNS has established two distinct categories under the umbrella of "causing death by negligence."
• The first category addresses causing death through any rash or negligent act that does not amount to culpable
homicide.
• Offenders in this category may face imprisonment for up to five years and a fine.
• The second category deals with causing death through rash and negligent driving, not amounting to culpable homicide.

For More Details Contact Us On: 0422-4954645; 9958604645


SA IAS ACADEMY UPSC (15/03/2024)
6. Consider the following statements regarding the Direct-to-Mobile (D2M) technology:
1. It is a combination of broadband and broadcast that uses mobile phones to capture territorial digital TV signals.
2. It can be used in emergency alert warning purposes without depending on internet/cellular networks.
Which of the statements given above is/are correct?
(a) 1 only
(b) 2 only
(c) Both 1 and 2
(d) Neither 1 nor 2
Answer C
Explanation:
Indian telecom operators have asked the government to auction the spectrum that will be used for direct-to-mobile
(D2M) technology services.
• The science behind D2M is similar to that of an FM radio, where a receiver within the device can tap into different radio
frequencies.
• This new-age technology is a combination of broadband and broadcast that uses mobile phones to capture territorial
digital TV signals.
• Using D2M, multimedia content, including live TV matches, can be streamed to phones directly without using the
internet.
D2M can ensure-
• Emergency alerts are delivered directly, reliably and without dependence on internet/cellular networks.
• Disaster Management audio content is delivered directly and authentically in a targeted manner.
With D2M, governments can broadcast citizen-centric information.

7. Consider the following statements regarding the Warli Tribes:


1. They mainly live in the bordering regions of Karnataka and Tamil Nadu.
2. They celebrate a festival called Bohada.
Which of the statements given above is/are correct?
(a) 1 only
(b) 2 only
(c) Both 1 and 2
(d) Neither 1 nor 2
Answer: B
Explanation:
Indigenous Warli Tribe, living near Sanjay Gandhi National Park in Maharashtra teaches a lesson about peaceful
coexistence with leopards.
• They are an adivasi indigenous tribe who live in the mountainous, coastal, and bordering regions of Gujarat and
Maharashtra.
The word 'Warli' is derived from the word 'Warla,' which means 'piece of land'.
• Language - The Warli people speak Varli or Warli, an Indo-Aryan language. The language is typically classified as
Marathi, but it is also known as Konkani or Bhil.
• Culture - They have their own animistic beliefs, way of life, customs, and traditions, and they have adopted many Hindu
beliefs.
• The Warli culture is centred on the concept of Mother Nature, and natural elements are frequently depicted as focal
points in Warli painting.

8. Consider the following statements regarding the Genocide Convention 1948:


1. Under it, an act of imposing measures intended to prevent births within the group is called as genocide.
2. India has not ratified this convention.
Which of the statements given above is/are correct?

For More Details Contact Us On: 0422-4954645; 9958604645


SA IAS ACADEMY UPSC (15/03/2024)
(a) 1 only
(b) 2 only
(c) Both 1 and 2
(d) Neither 1 nor 2
Answer : A
Explanation:
Recently, South Africa moved the International Court of Justice (ICJ), for an urgent order declaring that Israel was in
breach of its obligations under the 1948 Genocide Convention.
• The term 'genocide' is often loosely used when speaking of attacks against various communities across the world.
• It has been defined using set criteria in the UN's Convention on the Prevention and Punishment of the Crime of
Genocide, moved in the General Assembly in 1948.
• It says, "In the present Convention, genocide means any of the following acts committed with intent to destroy, in
whole or in part, a national, ethnical, racial or religious group, as such:
Killing members of the group;
• Causing serious bodily or mental harm to members of the group;
• Deliberately inflicting on the group conditions of life calculated to bring about its physical destruction in whole or in
part;
• Imposing measures intended to prevent births within the group;
Forcibly transferring children of the group to
another group.
• As per this convention the genocide is a crime whether committed during wartime or peacetime.
• India ratified the convention in 1959; there is no legislation on the subject.

9. The Lal Bahadur Shastri Institute of Technology for Women (LBSITW), which was recently in news for launching its
nanosatellite called WESAT, is located in which city?
(a) Bengaluru
(b) Pune
(c) Thiruvananthapuram
(d) Mangalore
Answer: C
Explanation:
On January 1, 2024, a nanosatellite called WESAT (Women Engineered SATellite) was launched by students of Lal Bahadur
Shastri Institute of Technology for Women in Thiruvananthapuram. The satellite was launched as secondary payload
aboard an Indian Space Research Organisation (ISRO) mission, specifically the Polar Satellite Launch Vehicle-C58/XPoSat
mission.
WESAT is a 1.4-kg satellite designed and developed entirely by women, making it the first Indian satellite with this
distinction. The purpose of WESAT is to study ultraviolet radiation and its impact. The satellite project was led by Lizy
Abraham, an Assistant Professor and Principal
Investigator, with technical assistance from ISRO.

10. With reference to Graphene, consider the following statements:


1. It is a one-atom-thick layer of carbon atoms arranged in a hexagonal lattice.
2. It is almost perfectly transparent and is impermeable to gases.
3. It is a poor conductor of heat and electricity.
How many of the above statements are correct?
(a) Only one
(b) Only two
(c) All three
(d) None

For More Details Contact Us On: 0422-4954645; 9958604645


SA IAS ACADEMY UPSC (15/03/2024)
Answer: B
Explanation:
Researchers recently created the world's first functional semiconductor made from graphene.
About Graphene:
• Graphene is a one-atom-thick layer of carbon atoms arranged in a hexagonal lattice.
• It is the building-block of Graphite (which is used, among others things, in pencil tips).
• It was first isolated in 2004.
• Properties:
• Graphene is the world's thinnest material; it is only one atom thick, one million times thinner than human hair.
• However, it is very strong, stronger than steel and diamond.
• It is an excellent conductor of heat and electricity. It conducts electricity better than copper.
• It is almost perfectly transparent, as it absorbs only 2% of light.
• It is impermeable to gases, even those as light as hydrogen and helium.

11. C- CARES web portal is associated with which sector?


(a) Petroleum sector
(b) Renewable energy sector
(c) Coal sector
(d) Agriculture sector
Answer: C
Explanation:
Recently, Union Minister Shri Pralhad Joshi launched the C-CARES web portal for the Coal Mines Provident Fund
Organization (CMPFO). Developed by C-DAC, it aims to digitize records and streamline processes for 3.3 lakh Provident
Fund subscribers and 6.1 lakh pensioners in the coal sector. The portal enables online settlement of claims, ensuring
transparency, faster processing, and better record management. Aligned with the vision of Digital India, it signifies a
significant leap towards efficient and transparent operations in the coal sector.

12. Consider the following statements regarding NHPC Limited:


1. It is mandated to develop hydroelectric power in the country.
2. It works under the Ministry of New and Renewable Energy (MNRE), Government of India.
Which of the statements given above is/are correct?
(a) 1 only
(b) 2 only
(c) Both 1 and 2
(d) Neither 1 nor 2
Answer: A
Explanation:
NHPC has inked an initial pact to invest Rs 4,000 crore in 750 MW Kuppa Pumped Hydro Storage Project at Chhota
Udaipur in Gujarat.
• NHPC Limited (formerly known as National Hydroelectric Power Corp.) is a Government of India Mini Ratna Category-l
Public Sector Enterprise under the Ministry of Power.
It was incorporated in the year 1975 under the Companies Act, 1956, with the objective of developing hydroelectric
power in the country.

13. In comparison to the typical day-night cycle on Earth, how long is the daylight period experienced by astronauts on
the International Space Station?
(a)12 hours
(b) 45 minutes
(c) 1 hour

For More Details Contact Us On: 0422-4954645; 9958604645


SA IAS ACADEMY UPSC (15/03/2024)
(d) 3 hours
Answer: B
Astronauts on the International Space Station experience 45 minutes of daylight followed by 45 minutes of darkness in
each orbit. This is significantly different from the typical 12-hour day-night cycle on Earth. Thus, the crew at International
Space Station witnesses 16 New Year countdowns in a row.

14. Consider the following statements regarding Aspergillus fumigatus:


1. It is a species of virus that causes diseases in humans.
2. It is found only in freshwater bodies.
Which of the statements given above is/are correct?
(a) 1 only
(b) 2 only
(c) Both 1 and 2
(d) Neither 1 nor 2
Answer: D
Explanation:
An international team of researchers recently unveiled ground-breaking findings on the fungus Aspergillus fumigatus,
which can cause deadly disease in humans.
About Aspergillus fumigatus:
It is a species of fungus that causes diseases in humans.
It can be found throughout the environment, including in soil, plant matter, and household dust.

15. Consider the following statements regarding the PRERANA program:


1. It is a week-long residential program for school students.
2. It aims to engage students in diverse indigenous knowledge systems and latest State-of-Art technologies.
Which of the statements given above is/are correct?
(a) 1 only
(b) 2 only
(c) Both 1 and 2
(d) Neither 1 nor 2
Answer:C
Explanation:
Recently, the Department of School Education & Literacy, Ministry of Education, Government of India has launched
'Prerana: An Experiential Learning program'.
• It aims to offer a meaningful, unique, and inspiring experience to all participants, thereby empowering them with
leadership qualities.

16. One Stop Centre scheme’ is formulated by which ministry?


(a) Ministry of Women and Child Development
(b) Ministry of Finance
(c)Ministry of Rural Development
(d) Ministry of Agriculture
Answer: A
Explanation:
The Union Minister for Women and Child Development announced the establishment of One Stop Centres (OSCs) in over
700 districts nationwide, under a Centrally Sponsored Scheme by the Ministry of Women and Child Development. The
initiative aims to offer comprehensive support for women facing violence, providing immediate access to medical, legal,
psychological, and counseling services. The funding comes from the Nirbhaya Fund, with 100% financial assistance from
the Central Government.

For More Details Contact Us On: 0422-4954645; 9958604645


SA IAS ACADEMY UPSC (15/03/2024)
17. Which are the primary objects of study for the recently launched XPoSat (X-ray Polarimeter Satellite) by ISRO?
(a) Exoplanets
(b) Neutron stars and Blackholes
(c) Distant stars and Solar system
(d) Asteroids
Answer: B
Explanation:
The primary objectives of the recently launched XPoSat (X-ray Polarimeter Satellite) by ISRO are to investigate the
polarization of intense X-Ray sources and study the enigmatic world of black holes. This satellite is India’s first dedicated
scientific satellite for conducting research in space-based polarization measurements of X-ray emission from celestial
sources.
It carries two scientific payloads: POLIX (Polarimeter Instrument in X-rays) for measuring polarimetry parameters in the
medium X-ray energy range of 8-30 keV photons and XSPECT (X-ray Spectroscopy and Timing) for providing spectroscopic
information in the energy range of 0.8-15 keV. These instruments will help in gaining a deeper understanding of X-ray
sources in space, including neutron stars and black holes.

18. With reference to Red weaver ants, consider the following statements:
1. These are recognized as bio-control agents.
2. They protect a variety of tropical crops against insect pests.
3. The chutney made with these ants has received a Geographical Identification (GI) tag.
How many of the statements given above are correct?
(a) Only one
(b) Only two
(c) All three
(d) None
Answer: C
Explanation:
Recently, the Similipal kai chutney made with red weaver ants by the tribal people of Odisha's Mayurbhanj district
(Odisha) received the geographical identity tag.
• The savoury chutney is popular in Mayurbhanj strong.
Key facts about red weaver ants
• These are indigenous to Mayurbhanj and are found in abundance in the jungles of every block area of the district,
including in the Similipal Tiger Reserve, throughout the year.
• They feed on small insects and other invertebrates like beetles, flies and hymenopterans.
• They do not sting but have a painful bite into which they can secrete irritant chemicals from their abdomens,
• They are also recognised as bio-control agents because they are aggressive and will prey on most arthropods entering
their territory.
• They protect a variety of tropical crops against insect pests, acting as an alternative to chemical insecticides.
Hence all statements are correct.

19. Consider the following statements regarding the Wetland City Accreditation:
1. It is given to cities which have taken exceptional steps to safeguard their urban wetlands.
2. No Indian city has received this recognition.
Which of the statements given above is/are correct?
(a) 1 only
(b) 2 only
(c) Both 1 and 2
(d) Neither 1 nor 2

For More Details Contact Us On: 0422-4954645; 9958604645


SA IAS ACADEMY UPSC (15/03/2024)
Answer: C
Explanation:
Recently, the Ministry of Environment, Forest and Climate Change has submitted three nominations from India for
Wetland City Accreditation (WCA) of Indore (Madhya Pradesh), Bhopal (Madhya Pradesh) & Udaipur (Rajasthan) under
the Ramsar Convention on Wetlands.
• The Ramsar Convention during COP12 held in the year 2015 approved a voluntary Wetland City. No india city as been
choices here.

20. What is ‘eROSITA’?


(a) Drone
(b) X-ray telescope
(c) Artificial Intelligence tool
(d) Submarine
Answer: B
Explanation:
The German eROSITA consortium has released data from the first all-sky survey conducted by the eROSITA X-ray
telescope on the Russian-German SRG observatory. eROSITA stands for extended ROentgen Survey with an Imaging
Telescope Array and is the main instrument on the Russian spacecraft Spektrum-Roentgen-Gamma (SRG). Launched in
July 2019, it maps the entire celestial sphere every six months.

21. Recently, which oceanographic research vessel of the Indian Navy has embarked on the Sagar Maitri Mission-4 to
Oman?
A. INS Makar
B. INS Sandhayak
C. INS Sagardhwani
D. INS Dhruv
Answer: C
Explanation:
INS Sagardhwani, an oceanographic research vessel of the Indian Navy, has embarked on the Sagar Maitri Mission-4 to
Oman. The goal of the mission is to establish long-term scientific partnerships and collaboration with Indian Ocean Rim
countries for ocean research and development. It will also showcase India’s capabilities in marine science and technology.

22. Consider the following statements regarding the Northeast African Cheetah:
1. It is mainly found in the Horn of Africa region.
2. It is categorized as critically endangered species under the IUCN Red list.
Which of the statements given above is/are correct?
A 1 only
B 2 only
C Both 1 and 2
D Neither 1 nor 2
Answer: A
Explanation:
A group of experts have appealed to the International Union for Conservation of Nature (IUCN) to reclassify the status of
the Northeast African Cheetah (Acinonyx jubatus soemmeringii) to 'endangered' from 'vulnerable'.
• It is found in the Horn of Africa. It is in vulnerable category of IUCN.

23. Consider the following statements regarding the National Real Estate Development Council (NAREDCO):
1. It is an industry association for the real estate sector in India.
2. It works under the aegis of the Union Ministry of Finance.

For More Details Contact Us On: 0422-4954645; 9958604645


SA IAS ACADEMY UPSC (15/03/2024)
Which of the statements given above is/are correct?
A 1 only
B 2 only
C Both 1 and 2
D Neither 1 nor 2
Answer: A
Explanation:
Real estate body NAREDCO recently said it plans to organise a builders' conference in Ayodhya to tap its commercial and
residential opportunities. It is under the ministry of housing affairs.

24. With reference to Fuel Cells, consider the following statements:


1. It is a device that generates electricity through a chemical reaction.
2. It has lower or zero emissions compared to combustion engines.
3. It can supply electrical energy for a shorter period of time when compared to batteries.
How many of the above statements are correct?
A Only one
B Only two
C Only three
D All four
Answer: B
Explanation:
ISRO recently said it has successfully tested a futuristic fuel cell-based power system.
About Fuel Cell:
• A fuel cell is a device that generates electricity by a chemical reaction.
• Fuel cells can be used in a wide range of applications, providing power for applications across multiple sectors, including
transportation, industrial/ commercial/residential buildings, and long-term energy storage for the grid in reversible
systems.
• Working:
• A fuel cell consists of two electrodes-a negative electrode (or anode) and a positive electrode (or cathode).
Both electrodes must be immersed in and separated by an electrolyte, which may be a liquid or a solid but must, in either
case, conduct ions between the electrodes in order to complete the chemistry of the system.
• A fuel, such as hydrogen, is supplied to the anode, where it is oxidized, producing hydrogen ions and electrons.
• An oxidizer, such as oxygen, is supplied to the cathode, where the hydrogen ions from the anode absorb electrons from
the latter and react with the oxygen to produce water.
• The difference between the respective energy levels at the electrodes (electromotive force) is the voltage per unit cell.

25. Who has been recently appointed as the Chairman of the Sixteenth Finance Commission?
A. Viral Acharya B. Jagdish Bhagwati
C. Arvind Panagariya D. Amit Mitra
Answer: C
Explanation:
Dr. Arvind Panagariya has been appointed as the new Chairman of the Finance Commission of India. The Finance
Commission is a constitutional body that gives recommendations on center-state financial relations. Dr. Panagariya is an
eminent economist who has served as the first Vice Chairman of NITI Aayog. He will now head the 16th Finance
Commission, which will suggest the tax devolution formula between the centre and states for 5 years starting 2026-27.

26. Consider the following statements regarding the Pallas fish eagle:
1. It is mainly found in the Central Asian Countries.
2. It is usually seen in desert areas.

For More Details Contact Us On: 0422-4954645; 9958604645


SA IAS ACADEMY UPSC (15/03/2024)
Which of the statements given above is/are correct?
A 1 only
B 2 only
C Both 1 and 2
D Neither 1 nor 2
Answer: A
Explanation:
After 10 years, the Pallas fish eagle, was sighted in the Chilika during the bird census carried out by the Chilika wildlife
division.
• It is also known as Pallas's sea eagle or band-tailed fish eagle, is a large, brownish sea eagle.
• Distribution: It is found in east Palearctic in Kazakhstan, Russia, Tajikistan, Turkmenistan, Uzbekistan, Mongolia, China,
India, Nepal, Bangladesh and Myanmar.

27. ‘Economics of the Food System Transformation’ report is released by which organization?
A. International Monetary Fund
B. Food and Agricultural Organization
C. Food System Economics Commission (FSEC)
D. World Bank
Answer: C
Explanation:
The 'Economics of the Food System Transformation' report is a global policy report from the Food System Economics
Commission (FSEC). The report estimates that transforming the global food system could create economic benefits of 5–
10 trillion USD per year. The report also states that the policies and implementation to achieve these transformations
would cost only 0.2–0.4% of global GDP. The report discusses the unsustainable trajectory of the global food system and
the potential economic benefits of a transformation. It also states that transforming food systems worldwide could help
address global climate, nature, and health emergencies.

28. Consider the following statements regarding the Rejupave technology:


1. It is developed by the CSIR-Central Road Research Institute.
2. It is used in constructing high-altitude bituminous roads at low temperature conditions.
Which of the statements given above is/are correct?
A 1 only
B 2 only
C Both 1 and 2
D Neither 1 nor 2
Answer: C
Explanation:
Recently, the Border Roads Organisation (BRO) has utilised road construction technology i.e Rejupave technology to build
high-altitude bituminous road sections at the Sela tunnel and LGG-Damteng-Yangste (LDY) road near the India-China
border in Arunachal Pradesh.
• It is developed by India's oldest and premier road research organisation, CSIR-Central Road Research Institute (CSIR-
CRRI).
It is beneficial in constructing high-altitude bituminous roads at low and sub-zero temperature conditions.

29. Where is the National Fisheries Development Board establishing an Eri silk spinning plant?
A. Guwahati, Assam
B. Mushalpur, Assam
C. Shillong, Meghalaya
D. Imphal, Manipur

For More Details Contact Us On: 0422-4954645; 9958604645


SA IAS ACADEMY UPSC (15/03/2024)
Answer: B
Explanation:
The National Fisheries Development Board is establishing an Eri silk spinning plant at Mushalpur, Baksa district in Assam.
This ₹15 crore project, part of the blue revolution scheme, will provide direct employment to 375 individuals and benefit
25,000 households engaged in silk production.

30. Consider the following statements regarding the Plasma waves:


1. These are only observed in outer space.
2. These are identified as the short-time scale fluctuations in the electric and magnetic field observations.
Which of the statements given above is/are correct?
A 1 only
B 2 only
C Both 1 and 2
D Neither 1 nor 2
Answer: B
Explanation:
Scientists have detected existence of high-frequency plasma waves in the Martian Upper Atmosphere with novel
narrowband and broadband features that can help to understand plasma processes in the Martian plasma environment.
• These waves are often observed in the Earth's magnetosphere, a magnetic field cavity around the
Earth.
• In general, plasma waves are identified as the short-time scale fluctuations in the electric and magnetic field
observations.
• These plasma waves play an important role in the energization and transport of the charged particles in the Earth's
magnetosphere.
• Some of the plasma waves like electromagnetic ion cyclotron waves act as a cleaning agent for the Earth's radiation
belt, which is hazardous to our satellites.
• Knowing this scenario, researchers are curious to understand the existence of various plasma waves in the vicinity of
unmagnetized planets like Mars.
• The planet Mars do not have any intrinsic magnetic field therefore the high-speed solar wind coming from the Sun
interacts directly with the Mars atmosphere, like an obstacle in the flow.
Hence only statement 2 is correct.

31. Consider the following statements regarding the recent RBI Norms for Politically-Exposed Persons (PEPs):
1. PEPs are individuals who are or have been entrusted with prominent public functions by a foreign country.
2. The RBI regulated entities (RES) are not allowed to transact with PEPs.
Which of the statements given above is/are correct?
A 1 only
B 2 only
C Both 1 and 2
D Neither 1 nor 2
Answer: A
Explanation:
The Reserve Bank of India (RBI) recently updated Know Your Customer (KYC) norms for politically exposed persons (PEPs)
who transact with regulated entities (RES), seeking to comply with the recommendations of the Financial Action Task
Force (FATF).
New RBI Norms for Politically-Exposed Persons (PEPs):
• Who are PEPs? In the amended KYC master direction, the central bank defines PEPs as "individuals who are or have
been entrusted with prominent public functions by a foreign country, including the heads of states/governments, senior

For More Details Contact Us On: 0422-4954645; 9958604645


SA IAS ACADEMY UPSC (15/03/2024)
politicians, senior government or judicial or military officers, senior executives of state-owned corporations, and
important political party officials".
• REs have the option of establishing a relationship with PEPs (whether as customers or beneficial owners).
• REs have to perform regular customer due diligence and also follow additional conditions prescribed by the RBI to
transact with PEPs.
• Some additional conditions include establishing an appropriate risk management system to determine whether the
customer or the beneficial owner is a PEP.
• REs have to take reasonable measures to establish the source of funds/ wealth.
• They also need to get approval from senior management to open an account for a PEP.

32. Consider the following statements regarding Laokhowa and Burhachapori Wildlife Sanctuaries:
1. They are Protected Areas (PAs) in Arunachal Pradesh.
2. They are located on the southern bank of the river Brahmaputra.
Which of the statements given above is/are correct?
A 1 only
B 2 only
C Both 1 and 2
D Neither 1 nor 2
Answer: B
Explanation:
Two rhinos have recently returned to the Laokhowa and Burhachapori Wildlife Sanctuary after almost a 40-year gap
following a successful anti-encroachment operation.
About Laokhowa and Burhachapori Wildlife Sanctuaries:
• Location:
• The Laokhowa and Burhachapori Wildlife Sanctuaries are two centrally located Protected Areas (PAs) of Assam.
• They are located on the southern bank of the river Brahmaputra.
• In fact, though these two wildlife sanctuaries have two different names, they are eologically and geographically a
singular entity.

33. What is the primary objective of the North-East Venture Fund set up by NEDFi?
A. To provide financial assistance for large infrastructure projects only.
B. To invest in real estate development across India.
C. To support and invest in startup ventures in the North-East region of India.
D. To provide consultancy services to state governments in the North-East.
Answer: C
Explanation:
North-East Venture Fund (NEVF), established by NEDFi (North Eastern Development Finance Corporation Ltd.), is aimed
at encouraging and collaborating with startup ventures in the North-East region of India.
With a commitment of ₹100 crore to the fund, contributed by the ministry, NEDFi, and SIDBI (Small Industries
Development Bank of India), NEVF focuses on fostering entrepreneurship and business initiatives in the region. This
initiative aligns with NEDFi’s broader role in providing financial assistance and advisory services for various enterprises
and projects in India’s North-East.

34. Consider the following statements regarding Indrayani River:


1. It is a tributary of the Bhima River.
2. It originates in the Western Ghats in Karnataka.
Which of the statements given above is/are correct?
A 1 only
B 2 only

For More Details Contact Us On: 0422-4954645; 9958604645


SA IAS ACADEMY UPSC (15/03/2024)
C Both 1 and 2
D Neither 1 nor 2
Answer: A
Explanation:
The recent reappearance of toxic foam on the Indrayani river once again raised concerns about pollution here.
About Indrayani River:
• It is a tributary of the Bhima River, which again is a tributary of the Krishna River. It originates in Kurvande village near
Lonavla, a hill station in the Sahyadri mountains of Maharashtra.
• Fed by rain, it flows east from there to meet the Bhima River.
• It follows a course mostly north of the city of Pune.
• The river has great religious importance, and the two sacred towns of Alandi and Dehu are situated on its banks.

35. Recently, which department launched the ‘StartupShala’, a flagship accelerator program of startup India?
A. Department of Agriculture and Farmers Welfare
B. Department for Promotion of Industry and Internal Trade
C. Department of Economic Affairs
D. Department of Agricultural Research and Education
Answer: B
Explanation:
StartupShala, launched by the Department for Promotion of Industry and Internal Trade (DPIIT), features the Flagship
Accelerator Program. Designed to propel early-stage startups, the program aims to expedite their growth by furnishing
resources and opportunities.
Through a vast network of partners spanning incubation centers, educational institutions, and corporate entities, startups
benefit from diverse facilities and resources, elevating their journey to the next level of success.

36. With reference to the Tapioca plant, consider the following statements:
1. It is known for its tuberous roots.
2. It thrives best in a tropical, warm and humid climate.
3. It requires black soil for its growth.
How many of the statements given above are correct?
A Only one
B Only two
C All three
D None
Answer: B
Explanation:
The ICAR-Central Tuber Crops Research Institute (CTCRI) here has issued an advisory on feeding animals with parts of
cassava (tapioca) in view of the incident in Idukki where 13 cows died in a farm recently.
• It is a major horticulture crop cultivated on nearly 3 lakh hectares in Tamil Nadu, producing 60 lakh tonnes It is
cultivated throughout the tropical world for its tuberous roots, from which cassava flour, breads, tapioca, a laundry
starch, and an alcoholic beverage are derived.
• Climatic conditions required
• Soil: Any well-drained soil, preferably red lateritic loamy soil.
• It thrives best in a tropical, warm, humid climate
Rainfall: Well-distributed rainfall of over 100 cm per annum.
This crop can be cultivated upto an elevation of 1000 m.
• All parts of cassava/tapioca - leaves, stem, tuber and rind - contain the compounds called cyanogenic glucosides (CNGs),
that is, linamarin and lotaustralin which are hydrolysed by endogenous enzyme linamarase to acetone cyanohydrin which
may break down spontaneously liberating free hydrogen cyanide.

For More Details Contact Us On: 0422-4954645; 9958604645


SA IAS ACADEMY UPSC (15/03/2024)
37. What is the name of government scheme with the primary aim of providing financial assistance to tuberculosis (TB)
patients for their nutritional needs?
A. TB Poshan Yojana
B. Nikshay Poshan Yojana
C. TB-DBT
D. Akshay Poshan
Answer: B
Explanation:
Nikshay Poshan Yojana (NPY) was introduced by the government in April 2018 with the primary aim of providing financial
assistance to tuberculosis (TB) patients for their nutritional needs. Recognizing undernutrition as a significant risk factor
for TB, the scheme offers ₹500 per month as direct benefit transfer (DBT) to TB patients. This monetary support is
intended to help patients purchase food and maintain a nutritious diet throughout the duration of their TB treatment.

38. Consider the following statements regarding the Project Veer Gatha:
1. It is a joint initiative of the Ministry of Defence and Ministry of Education.
2. It aims to disseminate the details of bravery of the Gallantry Awardees among students.
Which of the statements given above is/are correct?
A 1 only
B 2 only
C Both 1 and 2
D Neither 1 nor 2
Answer: C
Explanation:
The third edition of Project 'Veer Gatha', as part of Republic Day celebrations, has witnessed an overwhelming pan India
response.
• It is a joint initiative of Ministry of Defence and Ministry of Education.

39. Consider the following statements regarding Lunar Gateway Station:


1. It is a component of NASA's Artemis program.
2. It will be the first space station ever to exist outside of Low Earth Orbit (LEO).
Which of the statements given above is/are correct?
A 1 only
B 2 only
C Both 1 and 2
D Neither 1 nor 2
Answer: C
Explanation:
The UAE recently announced its participation in developing a module on Nasa's Lunar Gateway Station alongside the USA,
Japan, Canada, and the European Union.
About Lunar Gateway Station:
• It is a primary component of NASA's Artemis program.
• Artemis intends to establish a long-term base on the Moon (Artemis base), and the Lunar Gateway will serve as a multi-
purpose outpost that orbits the Moon.
• The Gateway is a multinational project involving four of the International Space Station partner agencies: NASA, the
European Space Agency (ESA), Japan's Aerospace Exploration Agency (JAXA), and the Canadian Space Agency (CSA).
• Basically, the Gateway Station is similar to the International Space Station currently in low Earth orbit, but the Gateway
will orbit the Moon.
• Incidentally, the Gateway will be the first space station ever to exist outside of low Earth orbit, or LEO.

For More Details Contact Us On: 0422-4954645; 9958604645


SA IAS ACADEMY UPSC (15/03/2024)
• From the Gateway, NASA and international partners can provide essential support for long-term human presence on
the lunar surface, as well as launch additional missions for deep space exploration.
• Its flight path is a highly elliptical orbit, bringing it both relatively close to the Moon's surface and also far away, making
it easier to pick up astronauts and supplies from Earth, around a five-day trip.
• It will also offer a place to relay communications and act as a base for scientific research.

40. Recently, which neurotechnology company has embedded the first computer chip in a human brain?
A. Neuralink
B. Kernel
C. Blackrock Neurotech
D. Neurable
Answer: A
Explanation:
Neuralink is the first company to implant a computer chip in a human brain. Neuralink is a neurotechnology company
founded in 2016 by Musk and seven other scientists and engineers. The company's implant, known as the "Link," is a
small device that's surgically placed inside the human brain. Neuralink says its device can interpret neural activity so a
person can operate a computer or smartphone by simply thinking. The company ultimately hopes to use the brain-
machine interface to restore sensory and motor functions in people with disabilities, treat neurological conditions like
dementia and depression, and even enable telepathic communication.

41. Which state is host of the India-UAE joint military exercise ‘Desert Cyclone 2024’?
A. Uttar Pradesh
B. West Bengal
C. Rajasthan
D. Karnataka
Answer: C
Explanation
Rajasthan will host the India-UAE joint military exercise ‘Desert Cyclone 2024’. The inaugural edition of this bilateral drill
will be conducted from January 2nd to January 15th, 2024 in Rajasthan. The aim is to enhance interoperability between
the Indian Army and UAE forces by exchanging best practices in areas like urban operations. This comes under expanding
defense ties between the two nations who are strategic partners.

42. With reference to Dynamic Asset Allocation (DAA), consider the following statements:
1. It involves adjustment of the weights in a portfolio based on the overall market performance.
2. In DAA, portfolio managers enjoy a high degree of flexibility in their choice of investments.
3. Its transaction costs are very low when compared with a strategic asset allocation strategy.
How many of the above statements are correct?
A Only one
B Only two
C Only three
D All four
Answer: B
Explanation:
Asset management company (AMC) PPFAS Mutual Fund recently filed for an open-ended dynamic asset allocation
scheme with the capital market regulator, the Securities and Exchange Board of India (SEBI).
• Most of the funds in this category are invested and spread across various sectors, including equity funds, real estate,
stocks, and bonds.
• Under the dynamic allocation strategy, a portfolio manager assesses the current market conditions and the
performance of each asset class.

For More Details Contact Us On: 0422-4954645; 9958604645


SA IAS ACADEMY UPSC (15/03/2024)
He uses the results of the assessment to reduce the weights of assets with bad performance and increase the weights of
assets with strong performance.
• Generally, a dynamic strategy is used in reaction to existing risks and market downturns.
• Unlike the strategic asset allocation strategy, dynamic asset allocation does not involve a target mix of assets. Thus,
portfolio managers enjoy a high degree of flexibility in their choice of investments.
• Dynamic allocation requires active portfolio management. Therefore, the success of the strategy depends not only on
the market conditions but also on the portfolio manager's ability to make good investment decisions and to adequately
respond to changes in the market.

43. Agasthyagama edge belongs to which one of the following species?


A. Snake
B. Kangaroo lizard
C. Wolf Spider
D. Frog
Answer: B
Explanation:
Scientists recently discovered a new kangaroo lizard species, Agasthyagama edge, in the Western Ghats. Named after the
EDGE program, it's the second in the Agasthyagama genus. Found in Kulamavu, Idukki, it belongs to the Agamidae family.
These terrestrial reptiles, lacking a fifth toe for climbing, prefer leaf-covered areas. Averaging 4.3 cm in length, they are
fast runners feeding on small insects, and they hide within dry leaves to avoid predators.

44. Consider the following statements regarding Taurus Missile, recently seen in the news:
1. It is an air-to-surface cruise missile.
2. It is developed by the United States of America.
Which of the statements given above is/are correct?
A 1 only
B 2 only
C Both 1 and 2
D Neither 1 nor 2
Answer: A
Explanation:
Pressure is mounting on German Chancellor to give the green light for long-range Taurus missiles that would be a
significant boost to Kyiv's weapons arsenal for striking critical Russian assets.
About Taurus Missile:
Taurus KEPD 350, known as the 'bunker buster', is a Swedish-German long-range air-to-surface cruise missile.
• The high-precision stand-off guided missile system can penetrate through dense air defence systems and destroy hard
and deeply buried stationary and semi-stationary military targets on the ground.
• The missile attacks target bridges, ships in ports, runways, command, control and control centers, bunkers, port
facilities, and air base buildings.
• It is in service with the German (Luftwaffe) and Spanish Air Forces.
• Features:
• It has an overall weight of 1,400kg.
• It has a length of 16.7 feet, a wingspan of 6.7 feet, and diameter of 3.5 feet.
• The missile is constructed of modular pieces that can be assembled in different ways depending on the mission. Its
electronic systems are also modular.

45. Which city is the location of recently inaugurated India’s first Girls’ Military School?
A. Lucknow
B. Vrindavan

For More Details Contact Us On: 0422-4954645; 9958604645


SA IAS ACADEMY UPSC (15/03/2024)
C. Gurgaon
D. Faridabad
Answer: B
Explanation:
The first Girls’ Military School in India was inaugurated in Vrindavan, Mathura. This inaugural event was jointly conducted
by Chief Minister Yogi Adityanath and Defence Minister Rajnath Singh. The Samvid Gurukulam Girls Military School,
located within the Vatsalya Gram campus and established by Sadhvi Ritambhara, aims to provide education and training
to 870 female students. This initiative marks a significant step in extending military education traditionally reserved for
male students in India to female students as well.

46. With reference to Sponge, consider the following statements:


1. It is a living animal which is made of loosely arranged cells.
2. It provides home for many other animals, plants, and microorganisms in a mutual symbiotic relationship.
3. Unlike seaweed, it possesses remarkable resilience to climate change.
How many of the statements given above are correct?
A Only one
B Only two
C All three
D none of the above.
Answer: C
Explanation:
Warming ocean forced women in Zanzibar to switch from seaweed to climate-resilient sponge farming to stay afloat.
• Sponge farming is a relatively new business opportunity that does not harm the marine environment.
• A sponge is a living animal which is made of loosely arranged cells that surround a skeleton of fibres.
• The specialised cells nestled within thousands of tiny chambers, act as microscopic pumps, and tirelessly drawing water
into the sponge's body with their whip-like tails.
• Sponges provide homes for many other animals, plants, and microorganisms. In many cases, they all work together in a
mutual symbiotic relationship.
• Sea sponges exist in all oceans around the world and make up 20% of the global silicon biological sink.
• This unique pumping mechanism, which helps sponges extract nutrition and oxygen, also purifies the ocean water by
removing impurities, including sewage.
• Uniqueness: Sponges, unlike seaweed, possess remarkable resilience to climate change, require minimal maintenance,
and command premium market pricesprices.

47. Consider the following statements regarding the Innovations for Defence Excellence (iDEX):
1. It aims to cultivate an innovation ecosystem in the Defence and Aerospace sector in India.
2. It is entirely funded and managed by the Ministry of Defence.
Which of the statements given above is/are correct?
A 1 only
B 2 only
C Both 1 and 2
D Neither 1 nor 2
Answer: A
Explanation:
Innovations for Defence Excellence- Defence Innovation Organization (iDEX-DIO) is all set to participate in the tenth
edition of the Vibrant Gujarat Summit 2024 from 10 to 12 January 2024 at Gandhinagar, Gujarat.
• It is the flagship scheme of the Ministry of Defence, Govt of India launched in 2018.
• The objective of the scheme is to cultivate an innovation ecosystem in the Defence and Aerospace sector by
collaborating with startups, innovators, MSMEs, incubators, and academia.

For More Details Contact Us On: 0422-4954645; 9958604645


SA IAS ACADEMY UPSC (15/03/2024)
IDEX offers grants and support for R&D with significant potential for future adoption in Indian defence and aerospace.
It is currently engaged with around 400+ Startups and MSMEs.
• It is recognized as a game-changer in the defence ecosystem, iDEX has received the PM Award for Innovation in the
defence sector.
• Funding: It will be funded and managed by a 'Defence Innovation Organization (DIO)' which has been formed as a 'not
for profit' company as per the Companies Act 2013 for this purpose, by the two founder members i.e. Defence Public
Sector Undertakings (DPSUs) - HAL & BEL.

48. With reference to Maldives, consider the following statements:


1. It consists of a chain of small coral islands.
2. Its islands are protected by barrier reefs from the destructive effects of monsoons.
3. It has a presidential form of government.
How many of the above statements are correct?
A Only one
B Only two
C Only three
D All four
Answer: C
Explanation:
The Maldives government recently suspended three of its ministers after they shared offensive remarks against the
Indian Prime Minister on social media.
About Maldives:
• It is a low-lying island country in the north-central
• Its closest neighbours are India, about 600 kilometres north-east, and Sri Lanka, about 645 kilometres north-east.
Geography:
• It consists of a chain of about 1,200 small coral islands and sandbanks (some 200 of which are inhabited), grouped in
clusters, or atolls.
• The atolls have sandy beaches, lagoons, and a luxuriant growth of coconut palms, together with breadfruit trees and
tropical bushes.
• The islands extend more than 510 miles (820 km) from north to south and 80 miles (130 km) from east to west.
• None of the coral islands stand more than 1.8 metres (six feet) above sea level.
• Barrier reefs protect the islands from the destructive effects of monsoons.
• Arabic, Hindi, and English are also spoken.
Islam is the state religion.
Economy: It revolves mainly around tourism.
• Constitutional Framework: The constitution of the Maldives was adopted in 2008.
• The head of state and government is the president, assisted by a vice president and a cabinet.
• The president and vice president are directly elected by universal suffrage to a maximum of two five-year terms.

49. What is the name of Radar that is being developed by India for the Sukhoi-30 MKI fighter jet fleet?
A. Trinetra
B. Virupaksha
C. Rohini
D. Swati
Answer: B
Explanation:
India is developing an advanced Active Electronically Scanned Array (AESA) radar system called ‘Virupaaksha’ to upgrade
its fleet of Su-30MKI fighter jets. Being manufactured indigenously it would be the most capable radar equipping any
Sukhoi variant globally. This airborne radar uses electronic instead of mechanical scanning.

For More Details Contact Us On: 0422-4954645; 9958604645


SA IAS ACADEMY UPSC (15/03/2024)
50. “Bharat 5G Portal- an integrated portal” is launched by which ministry?
A. Ministry of Communication
B. Ministry of Finance
C. Ministry of Commerce and Industry
D. Ministry of Information and Broadcasting
Answer: A
Explanation:

The Secretary, Department of Telecommunications, Ministry of Communications, unveiled the "Bharat 5G Portal" during
'Bharat Telecom 2024.' This integrated platform focuses on quantum, 6G, IPR, and 5G domains. In collaboration with
PANIIT USA, it introduces the Future Tech-Experts registration portal to support India's Telecom ecosystem. The portal
serves as a centralized hub for quantum, IPR, PoCs/Pilot, 5G, and 6G, promoting innovation, collaboration.

51. Consider the following statements regarding the National Centre for Seismology (NCS):
1. It is the nodal agency for monitoring earthquake activity and conducting seismological research in India.
2. It works as an attached office of the Ministry of Science and Technology, Government of India.
Which of the statements given above is/are correct?
A 1 only
B 2 only
C Both 1 and 2
D Neither 1 nor 2
Answer: A
Explanation:
An earthquake of magnitude 3.9 hit Jammu and Kashmir's Kishtwar district recently, as reported by the National Centre
for Seismology.
About National Centre for Seismology (NCS):
• It is an attached office of the Ministry of Earth Sciences (MOES)..
• The NCS monitors earthquake activity all across the country through its 24x7 round-the-clock monitoring center for
better understanding of earthquake source processes and their effect on the cause of earthquake-safe society.
• It consists of various divisions:
• Earthquake Monitoring and Services
• Earthquake Hazard and Risk Assessments
• Geophysical Observation Systems
• It operates and maintains the NSN, or the National Seismological Network. The NSN consists of 153 seismological
observatories spread across the country.
• NSN is capable of recording earthquakes and events of magnitude (M) >= 2.5 in and around Delhi, M≥3.0 for the North
East (NE) region, M >= 3.5 in the peninsular and extra-peninsular areas, and M≥4.0 in border regions.

52. With reference to National Commission for Backward Classes (NCBC), consider the following statements:
1. It is a constitutional body.
2. It is consulted by State Governments on all major policy matters affecting the socially and educationally backward
classes.
3. Its recommendations are binding on the Government.
How many of the statements given above are correct?
(a)Only one
(b)Only two
(c)All three
(d) None of the above

For More Details Contact Us On: 0422-4954645; 9958604645


SA IAS ACADEMY UPSC (15/03/2024)
Answer : B
Explanation:
The National Commission for Backward Classes (NCBC) has raised serious objections to the West Bengal government's
recommendation to include 83 castes in the central list of Other Backward Classes (OBCS)
About National Commission for Backward Classes
(NCBC):
• NCBC was initially constituted by the Central Government by the National Commission for Backward Classes Act, 1993,
under the Ministry of Social Justice and Empowerment.
• It has been accorded constitutional status through "The Constitution (One Hundred and Second Amendment) Act,
2018", whereby Article 338B has been inserted, forming a Commission for the socially and educationally backward classes
to be known as the NCBC.
• The amendment inserted Article 338B, Article 342A, and Clause 26C in Article 366.
• Composition: The Commission consists of a Chairperson, a Vice-Chairperson, and three other Members in the rank and
pay of Secretary to the Government of India.
• The Chairperson, Vice-Chairperson and other Members of the Commission shall be appointed by the President by
warrant under his hand and seal.
• Functions: It shall be the duty of the Commission
to investigate and monitor all matters relating to the safeguards provided for the socially and educationally backward
classes under this Constitution or under any other law for the time being in force or under any order of the Government,
and to evaluate the working of such safeguards.

53. Which state government signed an MoU with Jakson Green for a green hydrogen project?
A. Tamil Nadu
B. Rajasthan
C. Madhya Pradesh
D. West Bengal
Answer: B
Explanation:
Jakson Green, engaged in renewable energy solutions, recently signed an MoU with the government of Rajasthan to set
up a green hydrogen & green ammonia project. This assumes significance as India targets becoming a global hub for
green hydrogen production and export by 2030. Jakson Green is exploring sites in Gujarat and Tamil Nadu too for
potential scale up.

54. With reference to SWAYAM Plus' Platform, consider the following statements:
1. It offer programmes to enhance employability of youths.
2. It provides content in Indian regional languages.
3. It is managed by the NITI Aayog.
How many of the statements given above is/are correct?
(a)Only one
(b) Only two
(c) All three
(d)None
Answer B
Explanation:
Recently, the Union Minister of Education and Skill Development and Entrepreneurship launched the 'SWAYAM Plus'
platform.
• It offers courses developed collaboratively with the industry.
• This platform aims to enhance employability of both college students and lifelong learners.

For More Details Contact Us On: 0422-4954645; 9958604645


SA IAS ACADEMY UPSC (15/03/2024)
• It will offer programmes in sectors like Manufacturing, Energy, Computer Science and Engineering/IT/ITES,
Management Studies, Healthcare, Hospitality and Tourism besides Indian Knowledge Systems.
• Objectives
• It primarily focuses on building an ecosystem for all stakeholders in professional and career development, including
learners, course providers, industry, academia, and strategic partners.
• It enables a mechanism that provides credit recognition for high-quality certifications and courses offered by the best
industry and academia partners.
• Reaching a large learner base by catering to learning across the country, with a focus on reaching learners from tiers 2
and 3 towns and rural areas.
• It also envisions bringing in features such as access to mentorship, scholarships and job placements as value-added
services in due course of time, thus building a digital ecosystem for learners to pursue upskilling and re-skilling at all
levels, namely certificate, diploma or degree.

55. With reference to Additional Tier-1 (AT - 1) Bonds, consider the following statements:
1. They are perpetual bonds with no maturity date.
2. They have a lower interest rate than other bonds.
3. They have a call option, which allows the banks to buy back the bonds from the investors.
How many of the statements given above are correct?
(a) Only one
(b)Only two
(c)All three
(d)None
Answer : B
Explanation:
The State Bank of India (SBI) is unlikely to utilize the enabling provision to raise nearly Rs 11,900 crore through additional
tier-I (AT-1) bonds due to pricing issues in the current fiscal.
About Additional Tier-1 (AT-1) Bonds:
• AT-1 bonds are perpetual bonds with no maturity date.
• Investors in these bonds do not get their principal back.
• However, the interest continues forever. AT-1 bonds have a higher interest rate than other bonds.
• Due to the perpetual nature of AT-1 bonds, these are often treated and viewed as equity, not debt.
• How are AT-1 Bonds Issued?
• AT-1 bonds are issued by banks in accordance with the directions of the Reserve Bank of India (RBI).
• Financial institutions usually issue such bonds to fulfil their capital adequacy requirements (CAR).
• CAR is an assessment of a bank's capital and its risk-weighted assets.
• Capital adequacy norms were formulated under the Basel III accord of 2009 after the credit crisis of 2008.
• The money raised through these bonds is kept aside as a shock absorber by the bank.

56. With reference to International Astronomical Union (IAU), consider the following statements:
1. It governs international professional astronomical activities worldwide.
2. Its individual members are nominated by national governments of each country.
3. It is the only organization recognized professionally for the naming of astronomical bodies.
How many of the statements given above are correct?
(a)Only one
(b)Only two
(c) All three
(d) None
Answer : B
Explanation:

For More Details Contact Us On: 0422-4954645; 9958604645


SA IAS ACADEMY UPSC (15/03/2024)
The International Astronomical Union recently confirmed the existence of three currently unnamed moons one around
Uranus and two orbiting Neptune.
About International Astronomical Union (IAU):
• The IAU is a senior body governing international professional astronomical activities worldwide.
• It was established in 1919 as the first of a series of international unions for the advancement of specific branches of
science.
• Its mission is to promote and safeguard the science of astronomy in all its aspects, including research, communication,
education, and development, through international cooperation.
• The IAU is made up of various divisions, commissions, and working groups representing the various areas of
astronomical research, teaching, and other endeavors.
• Membership:
• Its individual members are professional astronomers from all over the world, at the Ph.D. level and beyond, who are
active in professional research, education, and outreach in astronomy.
• The IAU also has junior members.
• IAU membership spans 92 countries. Out of those countries, 85 are National Members.
• IAU activities range from the definition of fundamental astronomical and dynamical constants.

57. What is the name of the mission launched by the Health Ministry to screen people for sickle cell disease?
A. National Sickle Cell Anaemia Mission
B. National Sickle Cell Anaemia Elimination Mission
C. National Sickle Cell Anaemia Eradication Mission
D. National Sickle Cell Anaemia Control Mission
Answer: B
Explanation:
The National Sickle Cell Anaemia Elimination Mission was launched by the Health Ministry on 1st July 2023 at Shahdol,
Madhya Pradesh to screen people across India for sickle cell disease. It aims to screen 7 crore people over 3 years,
focusing on tribal populations and other high prevalence areas across 17 states. The mission provides screening,
prevention and management of sickle cell anemia. Its goal is to eliminate the disease in India through early detection and
intervention.

58. Recently, which state government has signed a MoU with NTPC for green hydrogen and renewable energy
development?
A. Tamil Nadu
B. Rajasthan
C. Maharashtra
D. Haryana
Answer: B
Explanation:
NTPC Green Energy Limited (NGEL) signed a Memorandum of Understanding (MoU) with the Maharashtra government
for the development of green hydrogen and renewable energy. The MoU includes the development of green hydrogen
and its derivatives, such as green ammonia and green methanol, with a capacity of up to 1 million tons per year. The MoU
also includes the development of renewable energy projects and pumped storage projects of 2 GW.

59. Consider the following:


1. Prime Minister
2. Speaker of the Lok Sabha
3. Leader of Opposition in Lok Sabha
4. Chief Justice of India
5. Union Law Minister

For More Details Contact Us On: 0422-4954645; 9958604645


SA IAS ACADEMY UPSC (15/03/2024)
How many of the above are members of the selection committee for recommending the Chairperson and the Members
of the Lokpal?
(a)Only two
(b) Only three
(c)Only four
(d)All five
Answer: C
Explanation:
Former Supreme Court judge Justice Ajay Manikrao Khanwilkar was recently appointed as the chairperson About Lokpal:
• It is a statutory body established under the Lokpal and Lokayuktas Act 2013.
• Mandate: To inquire into allegations of corruption against certain public functionaries and for related matters.
• Organisational Structure:
• The Lokpal will consist of a chairperson and a maximum of eight members.
• The Chairperson should be either the former Chief Justice of India, or a former Judge of the Supreme Court, or an
eminent person who fulfils the eligibility criteria as specified.
• Out of the maximum eight members, half will be judicial members. The judicial member of the Lokpal should be either a
former Judge of the Supreme Court or a former Chief Justice of a High Court.
• A minimum of fifty per cent of the Members will be from SC/ST/OBC/Minorities and women.
• How are members appointed?
• The Chairperson and the Members are appointed by the President of India on the recommendation of a selection
committee composed of the Prime Minister as the Chairperson, the Speaker of Lok Sabha,

60. With reference to Mauritius, consider the following statements:


1. It is located off the eastern coast of Africa.
2. It follows parliamentary republic form of democracy.
3. Majority of its population is of Indo-Pakistani origin.
How many of the statements given above are correct?
(a)Only one
(b)Only two
(c) All three
(d) None
Answer: C
Explanation:
More than 3,000 people are stranded in Mauritius waters after the Indian Ocean Island nation stopped a ship belonging
to Norwegian Cruise Line Holdings from docking at its ports due to what it said was a health risk.
About Mauritius:
• It is an island country in the Indian Ocean, located off the eastern coast of Africa.
• Land:
• It lies about 500 miles (800 km) east of Madagascar in the Indian Ocean.
• Its outlying territories are Rodrigues Island, situated about 340 miles (550 km) eastward, the Cargados Carajos Shoals,
250 miles (400 km) northeastward, and the Agalega Islands, 580 miles (930 km) northward from the main island.
• Mauritius also claims sovereignty over the Chagos Archipelago (including Diego Garcia), some 1,250 miles (2,000 km) to
the northeast, although this claim is disputed by Britain.
• Relief:
• It is volcanic in origin and is almost entirely surrounded by coral reefs.
• The northern part is a plain that rises to a central plateau. The plateau is bordered by small mountains.
• Approximately two-thirds of the population is of Indo-Pakistani origin, most of whom are descendants of indentured
labourers brought to work in the sugar industry during the 19th and early 20th centuries.

For More Details Contact Us On: 0422-4954645; 9958604645


SA IAS ACADEMY UPSC (15/03/2024)
• About one-fourth of the population is Creole (of mixed French and African descent), and there are small numbers of
people of Chinese and Franco-Mauritian descent.
Religion: About half of the population is Hindu, about one-third is Christian.

61. Which animal has recently been declared the national symbol of Kyrgyzstan?
A. Snow leopard
B. Yak
C. Eagle
D. Horse
Answer: A
Explanation:
Kyrgyzstan has declared the snow leopard as its new national symbol. Snow leopards inhabit the Tian Shan Mountain
range in the country. They feature in Kyrgyz folklore and are seen as a symbol of greatness, nobility and resilience.
Conservation of the endangered species will be promoted under the new status.

62. With reference to Flue Cured Tobacco, consider the following statements:
1. It has high sugar content.
2. It has a very low levels of nicotine.
3. It does not contain natural Tannin.
How many of the statements given above are correct?
(a) Only one
(b)Only two
(c) All three
(d) None
Answer : A
Explanation:
Government of India permits sale of Flue Cured Virginia (FCV) Tobacco on Tobacco Boards auction platform and waives
off penalty on sale of excess production of registered growers and unauthorized production of unregistered growers in
Karnataka.
• Curing is a process by which the harvested tobacco leaf is made ready for the market.
• It is a well standardized process especially in FCV tobacco to achieve the desirable qualities in the cured leaf along with
the removal of moisture.
• There are three types of tobacco curing methods traditionally used: Air-Cured, Fire-Cured, and Flue-Cured.
• Each of the different curing methods results in a tobacco product that is distinguishable by both its nicotine content and
its aroma.
• Why is Tobacco cured?
• To create smoking tobacco, the tobacco leaves need to be cured, or dried out.
• The wet, green tobacco leaves of a tobacco plant initially contain too much moisture to catch fire.
• They also have higher chlorophyll content. By releasing a certain amount of chlorophyll from the leaves during the
drying out process, the natural tannins come out giving the smoked tobacco its flavor and scent.
• Key characteristics of Flu Cured Tobacco
• Produces primarily cigarette tobacco.

63. With reference to Ken River, consider the following statements:


1. It is one of the major rivers of the Bundelkhand region of central India.
2. It is a tributary of the Chambal River.
3. It flows through Madhya Pradesh and Uttar Pradesh.
How many of the statements given above are correct?
(a)Only one

For More Details Contact Us On: 0422-4954645; 9958604645


SA IAS ACADEMY UPSC (15/03/2024)
(b)Only two
(c)All three
(d)None
Answer:B
Explanation:
The Prime Minister in 'Mann Ki Baat' recently cited crocodiles in Ken River to hail how technology is being used
extensively for the conservation of wildlife in different parts of the country.
About Ken River:
• It is one of the major rivers of the Bundelkhand region of central India.
• It flows through two states, namely Madhya Pradesh and Uttar Pradesh.
• It is a tributary of the Yamuna River. The river is the last tributary of the Yamuna before the Yamuna joins the Ganga.
• Course:
• The river originates near the village of Ahirgawan on the north-west slopes of the Kaimur Range in the district of
Jabalpur, Madhya Pradesh.
• Crossing the Bijawar-Panna hills, the river cuts a 60 km long, and 150-180 m deep gorge. It is joined by several streams
in this gorge, making waterfalls.
• It travels a distance of 427 km and then merges with the Yamuna at Chilla village, near Fatehpur in Uttar Pradesh.
• It is known for its rare Sajhar or Dendritic Agate stone.
• Banda city is located on the banks of Ken River.
• Tributaries: The major tributaries of the Ken River are Bawas, Dewar, Kaith, Baink, Kopra, and Bearma.

64. With reference to Permafrost, consider the following statements:


1. It is any ground that has been frozen continuously for a minimum of two years.
2. It is made of a combination of soil, rocks, and sand that are held together by ice.
3. Permafrost regions are always covered in snow.
How many of the statements given above are correct?
A Only one
B Only two
C All three
D None
Answer : B
Explanation:
Thawing permafrost in the Arctic could release radon, a radioactive gas that has the potential to cause cancer, scientists
have warned.
About Permafrost:
• Permafrost is any ground-from soil to sediment to rock-that has been frozen continuously for a minimum of two years
and as many as hundreds of thousands of years.
• It can extend down beneath the earth's surface from a few feet to more than a mile, covering entire regions, such as
the Arctic tundra, or a single, isolated spot, such as a mountaintop of alpine permafrost.
• Although the ground is frozen, permafrost regions are not always covered in snow.
• Global Distribution:
• They can be found on land and below the ocean floor.
• It is found in areas where temperatures rarely rise above freezing.
• They are most common in regions with high mountains and in Earth's higher latitudes-near the North and South Poles.
• Permafrost is found almost exclusively in the far northern reaches and high elevations of the Northern Hemisphere, in
places like Siberia, Alaska, the Canadian Arctic, Greenland, and the Tibetan Plateau.
• Composition:
• Permafrost is made of a combination of soil, rocks, and sand that are held together by ice. The soil and ice in permafrost
stay frozen all year.

For More Details Contact Us On: 0422-4954645; 9958604645


SA IAS ACADEMY UPSC (15/03/2024)
• Near the surface, permafrost soils also contain large quantities of organic carbon-a material leftover from dead plants
that couldn't decompose or rot away due to the cold.
• Lower permafrost layers contain soils made mostly of minerals.
• A layer of soil on top of the permafrost does not stay frozen all year. This layer, called the active layer, thaws during the
warm summer months and freezes again in the fall.

65. What is the name of the new frog species discovered in Arunachal Pradesh named after the Patkai hills?
A. Arunchalops patkaiensis
B. Gracixalus patkaiensis
C. Rana patkaiensis
D. Kaloula patkaiensis
Answer: B
Explanation:
A new species of see-through green frog named Gracixalus patkaiensis has been discovered in the biodiverse Namdapha
National Park in Arunachal Pradesh. The tiny 2.2 cm frog inhabits dense undergrowth in evergreen forests at the foothills
of the Patkai range, and has a distinct insect-like call. It has been named after the Patkai hills of the Eastern Himalayas.

66. With reference to Floodplain, consider the following statements:


1. It is a generally flat area of land next to a river or stream.
2. It is composed of unconsolidated sedimentary deposits (alluvium).
3. Floodplains are absent where the downcutting of rivers is dominant.
How many of the statements given above are correct?
A Only one
B Only two
C All three
D None
Answer: C
Explanation:
The high-level committee for Yamuna's rejuvenation has set a target for the floodplain's demarcation.
About Floodplain:
• A floodplain is a generally flat area of land next to a river or stream.
• It is composed of unconsolidated sedimentary deposits (alluvium) and is subject to periodic inundation by the stream.
• It stretches from the banks of the river to the outer edges of the valley.
• A floodplain consists of two parts.
• The first is the main channel of the river itself, called the floodway. Floodways can sometimes be seasonal, meaning the
channel is dry for part of the year.
• Beyond the floodway is the flood fringe. It is the land between the banks of the floodway and the valley wall, or
anywhere the valley land starts to rise.
• How do floodplains form? Floodplains develop in two common ways: erosion and deposition (also known as
aggradation).

• When rivers start to meander, curving from side to side, the water erodes the banks of the river and creates a wide, flat
area around the sides.
• When the river floods, either because of heavy rainfall or ice melting upstream, it floods its banks.
• Floodplains are absent where downcutting of rivers is dominant.
• Importance:
The deposition of sediment that happens in floodplains can be the source of major fertility.
• This sediment is usually built up of alluvium, or silt, which is considered some of the richest soil, containing nutrients
like potash, phosphoric acid, and lime.

For More Details Contact Us On: 0422-4954645; 9958604645


SA IAS ACADEMY UPSC (15/03/2024)
67. According to Snow Leopard Population Assessment in India (SPAI) report, which state/UT has the highest number
of snow leopard?
A. Ladakh
B. Jammu and Kashmir
C. Himachal Pradesh
D. Sikkim
Answer: A
Explanation:
Union Minister of Environment, Forest, and Climate Change, Shri Bhupender Yadav released India’s first-ever Snow
Leopard Population Assessment report.
The four-year-long scientific exercise, coordinated by the Wildlife Institute of India, estimates approximately 718 snow
leopards in the wild. Ladakh leads with 477, followed by Uttarakhand (124), Himachal Pradesh (51), Arunachal Pradesh
(36), Sikkim (21), and Jammu and Kashmir (nine). India holds 10-15% of the global snow leopard population.

68. With reference to Section 354 of the Indian Penal Code (IPC), consider the following statements:
1. It punishes the use of criminal force on a woman with the intention of outraging her modesty.
2. It can be punished with a minimum imprisonment of ten years.
3. It does not encompass offences committed against men.
How many of the statements given above are correct?
A Only one
B Only two
C All three
D None
Answer B
Explanation:
The High Court of Jammu and Kashmir and Ladakh recently quashed the issuance of process by a trial court against a man
for the commission of the offence under Section 354 of the IPC for allegedly slapping his wife in public.
About Section 354 of IPC:
• Section 354 of the IPC states that anyone who assaults or uses criminal force against a woman with the intention of
outraging her modesty, or knowing that such an act is likely to outrage her modesty, can be punished with imprisonment
ranging from one to five years and may also be fined.
• It has been enacted with a view to protect a woman against indecent assault as well as to safeguard public morality and
decent behaviour.
• This section punishes an assault or use of criminal force on any woman with the intention or knowledge that the
woman's modesty will be outraged.
• The essential ingredients of the offense under Section 354 of the IPC are as follows:
that the person assaulted must be a woman
that the accused should have used some sort of criminal force on her
• Criminal force must have been used to outrage her modesty
• There must be an intention to do so. It is the section's substance, where the choice of punishment and the legitimacy of
the charges is evaluated.
• What constitutes an 'outrage to female modesty' has not been defined anywhere under the IPC. It is rather subject to
the facts and circumstances of each case.
• It does not encompass offences committed against men in the same manner.
• An offence under Section 354 of the IPC shall be a cognizable and a non-bailable offence that is triable by a magistrate
of any class.
• The Criminal Law Amendment Act of 2013 introduced four additional subsections to Section 354 of IPC to broaden its
scope. These are the following:

For More Details Contact Us On: 0422-4954645; 9958604645


SA IAS ACADEMY UPSC (15/03/2024)
• Section 354A: Sexual harassment and punishment for sexual harassment
• Section 354B: Assault or use of criminal force to woman with intent to disrobe
Section 354C: Voyeurism
Section 354D: Stalking.

69. Which UT has become the first Union Territory to implement PM Vishwakarma Yojana?
A. Ladakh
B. Jammu & Kashmir
C. Puducherry
D. Lakshadweep
Answer B
Explanation:
Jammu & Kashmir has become the first Union Territory to roll out the PM Vishwakarma Yojana skill training program for
craftsmen and artisans. 30 trainees have enrolled in the first batch focused on tailoring and dress design at the ITI center
in Shopian. Implementation aims to foster local businesses and self-employment.

70. Darlipali thermal power plant is located in which state?


A. Odisha
B. Gujarat
C. Karnataka
D. Madhya Pradesh
Answer A
Explanation:
Prime Minister Narendra Modi inaugurated NTPC Ltd's Darlipali thermal power plant (1,600 MW), Rourkela PP-II
expansion project (250 MW), and laid the foundation stone for Talcher 2x660 MW in Odisha, with a total investment of
Rs 28,978 crore. Darlipali, a pit-head station, has supply agreements with multiple states. The Rourkela project supports
the steel plant, while Talcher's 660 MW will cater 50% to Odisha and the rest to Tamil Nadu, Gujarat, and Assam.

71. Which one of the following best describes the term ‘Blue Economy 2.0’, seen in the budget 2024?
A. Sustainable development related to oceans, seas, and coasts
B. Providing New Agricultural Practice methods
C. Improving Urban Infrastructure
D. Renewable Energy Sources
Answer A
Explanation:
The Interim Budget prioritizes the advancement of 'Blue Economy 2.0,' concentrating on sustainable development
concerning oceans, seas, and coasts. Key initiatives include schemes for coastal restoration, aquaculture, and mariculture,
with an integrated approach.
Plans encompass setting up five integrated aquaparks and enhancing the Pradhan Mantri Matsya Sampada Yojana for
increased productivity, exports, and job creation. The Blue Economy is defined by the World Bank as the sustainable use
of ocean resources for economic growth and improved livelihoods, preserving ocean ecosystem health.

72. Consider the following:


1. Finland
2. Sweden
3. Norway
4. Croatia
How many of the above countries are member of the Nordic-Baltic cooperation?
A Only one

For More Details Contact Us On: 0422-4954645; 9958604645


SA IAS ACADEMY UPSC (15/03/2024)
B Only two
C Only three
D All four
Answer : C
Explanation:
The eight Nordic-Baltic countries are participating in Raisina Dialogue in New Delhi together as representatives of the
Nordic-Baltic cooperation.
• It is a regional cooperation format which as of 1992 has brought together five Nordic countries and three Baltic
countries in order to discuss important regional and international issues in an informal atmosphere.
• In 2000, it was decided that the Nordic-Baltic cooperation format would be called Nordic-Baltic Eight (NB8).
• Member countries: Finland, Sweden, Norway, Iceland, Denmark, Estonia, Latvia and Lithuania.
• Nordic countries are members of the European Union (except Iceland and Norway which are members of EFTA).
• Nordic countries collectively represent an economy of more than USD 2.012 trillion with a population of just above 27
million allowing a very high standard of living.
• India and NB8
• The Nordic-Baltic cooperation with India spans fields as diverse as innovation, green transition, maritime, health,
intellectual property rights, new technologies, space cooperation and artificial intelligence, student exchanges, culture
and tourism.
• Trade and investment figures between Nordic region and India are steadily increasing.

73. What is the primary purpose of the 'National Single Window System (NSWS)' launched by CDSCO?
A. Digital Health Records Management
B. Streamlining Medical Device Import
C. Medical Research Funding
D. Public Health Surveillance
Answer B
Explanation:
The 'National Single Window System (NSWS)' launched by India's Central Drugs Standard Control Organisation (CDSCO) is
primarily designed to streamline the import of medical devices. Developed by Tata Consultancy Services (TCS) through
Invest India, this unified portal aims to create a centralized platform for investors, thus enhancing the ease of doing
business. The NSWS serves as a one-stop-shop for all approvals related to the import of medical devices, facilitating
applications for certificates of registration, manufacturing licenses, and import licenses.

74. Consider the following:


1. India
2. Sri Lanka
3. Maldives
4. Myanmar
How many of the above countries participate in Exercise Dosti?
A Only one
B Only two
C Only three
D All four
Answer C
Explanation:
Indian and Sri Lankan coast guard ships reached the Maldives recently to take part in the trilateral coast guard exercise
Dosti 16.
About Exercise Dosti:

For More Details Contact Us On: 0422-4954645; 9958604645


SA IAS ACADEMY UPSC (15/03/2024)
• It is a trilateral coast guard exercise between India, Sri Lanka, and the Maldives.
• It is a biennial exercise.
• It was first conducted in 1991 between the Indian and Maldives Coast Guards.
• Sri Lanka joined the exercise for the first time in 2012. Dosti was last conducted in 2021.
• The exercises have focused on exercises and drills on providing assistance in sea accidents, eliminating sea pollution,
and the coast guard's procedures and conduct during situations such as oil spills.
• The aim of the exercise is to further fortify the friendship, enhance mutual operational capability, exercise
interoperability and build cooperation between the Coast Guards of India, Sri Lanka, and the Maldives.
• Dosti 16:
• It is the 16th edition of the exercise.
• The Coast Guards of the Maldives, India, and
Sri Lanka, along with observers from Bangladesh, is participating in Dosti 16 to enhance collaboration between the forces.
India is sending ICGS Samarth (with integral helo), ICGS Abhinav, and ICG Dornier for the exercise.

75. What is the purpose of the C-bot, an autonomous underwater vehicle, launched by the CSIR- NIO in Goa?
A. Space Exploration
B. Monitoring coral reefs
C. Agricultural Research
D. Air Quality Measurement
Answer B
Explanation:
The Goa-based CSIR-National Institute of Oceanography (NIO) launched C-bot, an autonomous underwater vehicle for
monitoring coral reefs. The vehicle is also known as the Coral Reef Monitoring and Surveillance Robot. C-bot is a robot
with advanced features for increased surveillance over the coral reefs. It can travel to a depth of 200 meters underwater.

76. With reference to Cantor's giant softshell turtle, consider the following statements:
1. It is endemic to India only.
2. It spends most of its life buried and motionless in sand.
3. It is listed as Critically Endangered under IUCN Red List.
How many of the statements given above are correct?
A Only one
B Only two
C All three
D None
Answer B
Explanation:
Recently, conservationists from the University of Portsmouth uncovered the nesting site of the "secretive" Cantor's giant
softshell turtle on the banks of the Chandragiri River in Kerala.
• It is also known as Asian giant softshell turtle and the frog-faced softshell turtle.
• It is a species known for its rarity and secretive nature.
• It spends most of its life buried and motionless, with only their eyes and mouth protruding from the sand.
• It surface only twice a day to take a breath and capture their prey by sit-and-wait strategy using an element of surprise.
• These turtles are primarily carnivores (piscivores) feeding on fish, crustaceans, and mollusks.
• Distribution: It is found in eastern and southern India, Bangladesh, Burma, Thailand, Malaysia, Laos, Cambodia,
Vietnam, eastern and southern China.
• Habitat: They inhabit inland, slow-moving, freshwater rivers, lakes, streams, and estuaries.
• Conservation status
• IUCN: Critically endangered
• CITES: Appendix II

For More Details Contact Us On: 0422-4954645; 9958604645


SA IAS ACADEMY UPSC (15/03/2024)
• Wildlife Protection Act, 1972: Schedule I
• Threats: Habitat destruction has made it disappear from much of its environment. They are also heavily.
• CITES: Appendix II
• Wildlife Protection Act, 1972: Schedule I
• Threats: Habitat destruction has made it disappear from much of its environment. They are also heavily harvested by
locals for meat.
Hence only statements 2 and 3 are correct.

77. Which three items from Arunachal Pradesh recently received the Geographical Indication (GI) tag?
A. Adi kekir, Wancho crafts, Changlang textiles
B. Apatani rice, adi kekir, Tibetan carpets
C. Adi kekir, Tibetan carpets, Wancho wooden crafts
D. Khamti rice, adi kekir, Changlang textiles
Answer C
Explanation:
Three products of Arunachal Pradesh – adi kekir ginger, carpets handmade by Tibetan settlers and wooden items by the
Wancho community – have received the prestigious GI tag recognizing their unique geographical roots. Adi kekir is a
famed variety of ginger while Wancho artisans create sculpture figures on wooden items. The GI tag certifies specific
products as originating from a particular region to boost their marketability.

78. With reference to the Committee of Privileges in Lok Sabha, consider the following statements:
1. It consists of 10 members elected by the members of the Lok Sabha from amongst themselves.
2. It examines every question involving a breach of privilege of the House or of the members.
3. The Speaker of Lok Sabha does not have the authority to pass final orders on its reports.
How many of the statements given above are correct?
A Only one
B Only two
C All three
D None
Answer : A
Explanation:
The Supreme Court recently stayed the proceedings of the Lok Sabha Privilege committee against the Chief Secretary,
Director General of Police and three other officials of the State of West Bengal.
About Committee of Privileges in Parliament:
• This committee consists of 15 members in Lok Sabha (10 in the case of Rajya Sabha) nominated by the Speaker
(Chairman in the case of Rajya Sabha).
• In the Rajya Sabha, the deputy chairperson heads the committee of privileges.
• Powers and Functions:
The committee examines every question involving a breach of privilege of the House, or of the members, or of any
Committee thereof referred to it by the House or by the Speaker/Chairman.
• It also determines, with reference to the facts of each case, whether a breach of privilege is involved and makes
suitable recommendations in its report.
• It also states the procedure to be followed by the House in giving effect to its recommendations.
• When a question of privilege is referred to the Committee by the House, the report of the Committee is presented to
the House by the Chairman or, in his absence, by any member of the Committee.
• Where a question of privilege is referred to the Committee by the Speaker, the report of the Committee is presented to
the Speaker, who may pass final orders thereon or direct that it be laid on the table of the House.
• The Speaker/Chairman may refer to the Committee any petition regarding the disqualification of a member on the
ground of defection for making a preliminary inquiry and submitting a report to him.

For More Details Contact Us On: 0422-4954645; 9958604645


SA IAS ACADEMY UPSC (15/03/2024)
• The procedure to be followed by the Committee in these cases is, so far as may be, the same as that applicable to
questions of breach of privilege.

79. With reference to Pigeonpea crop, consider the following statements:


1. It is a legume crop cultivated in semi-arid regions.
2. It requires rainy and moist conditions during its flowering phase.
3. Sandy loam soil is most suitable for its cultivation.
How many of the statements given above are correct?
A Only one
B Only two
C All three
D None
Answer: B
Explanation:
According to the International Crops Research Institute for the Semi-Arid Tropics (ICRISAT) a new fast-breeding protocol
is likely to make it easier for scientists to develop better quality varieties of the pigeonpea crop at a faster rate.
• It is also called as arhar and tur in India.
• It is an important legume crop and protein-rich food which is primarily consumed as dal in India.
• It is predominantly a crop of tropical areas mainly cultivated in semi arid regions of India.
• Climatic conditions
• Rain: It requires 600-650mm of annual rainfall along with moist conditions for the initial eight weeks and dry conditions
during its flowering and pod development phase.
• Temperature: It can be grown with a temperature ranging from 26 ^ 0 * C to 30 ^ 0 * C in the rainy season and 17 ^ 0 *
C to 22 ^ 0 * C in the post rainy (November to March) season.
• Soil: It can be grown on all types of soil, however, sandy loam or loam soil is most suitable for its cultivation.
• It is very sensitive to low radiation at pod development, therefore flowering during the monsoon and cloudy weather,
leads to poor pod formation.
• It is commonly intercropped with a wide range of crops. In India, it was estimated that 80-90% of the pigeonpea were
intercropped.
• The important diseases of Pigeon pea are Wilt, Sterility mosaic disease, Phytophthora blight, Alternaria blight and
Powdery mildew etc.
• Concern: The Pigeonpea's long growth cycle
and sensitivity to day length have hindered breeding efforts, with only about 250 varieties released globally over six
decades.
• Health benefits: It has a low glycaemic index and is rich in thiamine, riboflavin, niacin, vitamin B-6, folate, vitamin A,
calcium, zinc, iron, magnesium and phosphorus.
• Major Pigeon pea-producing states: Uttar Pradesh, Madhya Pradesh, West Bengal, Bihar and Jharkhand.

80. Consider the following statements regarding the Rashtriya Udyamita Vikas Pariyojana:
1. It focuses on reskilling and upskilling employees to enhance their competitiveness.
2. It provides training through offline mode only.
Which of the statements given above is/are correct?
A 1 only
B 2 only
C Both 1 and 2
D Neither 1 nor 2
Answer:A
Explanation:

For More Details Contact Us On: 0422-4954645; 9958604645


SA IAS ACADEMY UPSC (15/03/2024)
Recently, the union Minister of Education and Skill Development & Entrepreneurship inaugurated the Rashtriya Udyamita
Vikas Pariyojana, in Sambalpur, Odisha.
• It is tailored specifically for beneficiaries of the PM SVANidhi scheme, this unique National Entrepreneurship
Development Project underscores the government's steadfast commitment to nurturing job providers across the nation.
• Aim: It aims to equip individuals with comprehensive entrepreneurship training, creating job providers rather than job
seekers.
• This initiative focuses on reskilling and upskilling employees to enhance their competitiveness and adaptability in an era
of disruptive technology.
• It will offer comprehensive entrepreneurship training over a period of 22 weeks, combining theoretical knowledge with
practical exposure through experiential learning.
• The training will be conducted through offline, online and hybrid modes, with certificates awarded upon completion,
enhancing the course's credibility and value.
• The Government of India has partnered with Flipkart to skill street vendors and small shopkeepers.
• Under the pilot program, street vendors of 10 big cities of the country will be given a stipend along with training to
expand their businesses.

81. Which public sector space enterprise will launch India's GSAT-20 satellite aboard a SpaceX rocket later in 2024?
A. Indian Space Research Organisation
B. Hindustan Aeronautics Ltd
C. NewSpace India Limited
D. Antrix Corporation Ltd
Answer C
Explanation:
NewSpace India Limited, the commercial arm of ISRO, plans to launch communications satellite GSAT-20 in Q2 of 2024 on
SpaceX's Falcon 9 rocket. It is a demand-driven high throughput satellite ordered by NSIL to meet India's growing
broadband needs.

82. With reference to the Model Code of Conduct (MCC), consider the following statements:
1. It comes into effect from the time the EC announces the schedule of elections.
2. All the provisions of the MCC are legally enforceable.
3. During general elections to the Lok Sabha, MCC is applicable throughout the country.
How many of the statements given above are correct?
A Only one
B Only two
C All three
D None
Answer : B
Explanation:
An Indian delegation is set to leave for London in an effort to seal the free trade agreement with the UK, as dates for the
general elections are expected to be announced in less than a month, which will trigger the Model Code of Conduct
(MCC).
About Model Code of Conduct (MCC):
• MCC s a set of guidelines intended to regulate political parties and candidates during elections to ensure that the
conduct of polls by the Election Commission of India (ECI) is free and fair.
• This is something political parties and candidates voluntarily agree to.
• Divided into eight parts, the guidelines cover dos and don'ts for electioneering as well as on publication of
advertisements at the cost of the exchequer, among other things.
• The ECI ensures observance of the MCC by a political party in power and contesting candidates for conducting free and
fair elections under Article 324 of the Constitution of India.

For More Details Contact Us On: 0422-4954645; 9958604645


SA IAS ACADEMY UPSC (15/03/2024)
• The salient features of the Model Code of Conduct lay down how political parties, contesting candidates and party(s) in
power should conduct themselves during the process of elections i.e. on their general conduct during electioneering,
holding meetings and processions, poll day activities and functioning of the party in power etc.
• MCC also ensures that official machinery for electoral purposes is not misused. Further, it is also ensured that electoral
offenses, malpractices, and corrupt practices are prevented by all means. In case of violation, appropriate measures are
taken.
• The code comes into effect from the time the EC announces the schedule of elections and will be in force till the results
are announced.
• Applicability:
• During general elections to the Lok Sabha, the code is applicable throughout the country.
• During general elections to the Legislative Assembly of the state, the code is applicable in the entire State.
• During bye-elections, the MCC would be applicable in the area of the concerned Constituency only.
• Is MCC Legally Enforceable?
• The MCC by itself is not legally enforceable but certain provisions of the MCC may be enforced through invoking
corresponding provisions in other statutes such as the Representation of the People Act, 1951, IPC and CrPC.

83. Which Union Ministry implemented the 'Saksham Anganwadi and Poshan 2.0 scheme'?
A. Ministry of Railways B. Ministry of Women and Child Development
C. Ministry of Home Affairs D. Ministry of Rural Development
Answer: B
Explanation:
The budget allocation for Saksham Anganwadi and Poshan 2.0 in 2023-24 is ₹25,449 crore, which is a 6%, increase from
the revised estimates of ₹23,913 crore for 2022-23. In 2024, the budget estimate is ₹20,554 crore, which is 1.4% higher
than the revised allocation of ₹20,263 crore for 2023. Saksham Anganwadi and Poshan 2.0 is a part of the Mission Poshan
2.0, which was launched in February 2021 by Finance Minister. The Ministry of Women and Child Development is
responsible for implementing this scheme.

84. In the interim budget 2024-25, which of the following crops are taken into consideration for ‘Atmanirbhar Oil Seeds
Abhiyan’?
A. Mustard, Groundnut, Sesame, Soybean & Sunflower
B. Palm, Coconut, Canola & Castor
C. Peanuts, Cotton & Flax
D. Rapeseed & Palm
Answer A
Explanation:
Union Minister for Finance & Corporate Affairs Nirmala Sitharaman, highlighted farmer's welfare and rural demand in the
interim budget 2024-25. The budget emphasizes achieving 'atmanirbharta' in oil seeds, outlining strategies like research
for high-yielding varieties, modern farming techniques adoption, market linkages, procurement, value addition, and crop
insurance for mustard, groundnut, sesame, soybean, and sunflower.

85. Which organization recently signed an MoU with National Remote Sensing Centre to develop a Green Cover Index
for National Highways?
A. Ministry of Road Transport and Highways B. National Highways Authority of India
C. Bureau of Indian Standards
D. Defence Research and Development Organisation
Answer: B
Explanation:

For More Details Contact Us On: 0422-4954645; 9958604645


SA IAS ACADEMY UPSC (15/03/2024)
The National Highways Authority of India (NHAI) entered an MoU with ISRO's National Remote Sensing Centre (NRSC) for
3 years to develop a "Green Cover Index" using satellite imagery to monitor plantation along vast network of national
highways across India.

86. With reference to the Narcotic Drugs and Psychotropic Substances (NPDS) Act, 1985, consider the following
statements:
1. It prohibits any individual from the production of any narcotic drug or psychotropic substance.
2. It provides for the forfeiture of property derived used in illicit traffic in narcotic drugs and psychotropic substances.
3. A sentence awarded for the consumption of drugs under the NDPS Act is non-commutable.
How many of the statements given above are correct?
A Only one
B Only two
C All three
D None
Answer B
Explanation:
In a significant legal development, the Supreme Court recently overturned an order granting anticipatory bail to a
respondent accused under the Narcotic Drugs and
About Narcotic Drugs and Psychotropic Substances (NPDS) Act, 1985:
• The NDPS Act prohibits any individual from engaging in any activity consisting of the production, cultivation, sale,
purchase, transport, storage, and/ or consumption of any narcotic drug or psychotropic substance.
• Objectives:
• To take measures for preventing, combating, and regulating operations relating to narcotic drugs and psychotropic
substances.
• To provide for the forfeiture of property derived from or used in, illicit traffic in narcotic drugs and psychotropic
substances.
• To implement the provisions of the international conventions on Narcotic Drugs and Psychotropic Substances and for all
relevant matters.
• To add or omit the list of psychotropic substances.
• What are Narcotic Drugs and Psychotropic Substances?
• "Narcotic Drug" means coca leaf, cannabis (hemp), opium, popy straw, and includes all manufactured drugs.
• "Psychotropic substance" means any substance, natural or svnthetic. or any natural material or anv salt or preparation
of such substance or material included in the list of psychotropic substances specified in the Schedule.
• Applicability: The NDPS Act prohibits a person from manufacture / production / cultivation/ possession/ sale / purchase
/ transport/store/ consume any narcotic drug or psychotropic substance without due permission from the appropriate
authorities.
• Punishment under the NDPS Act:
• The Act follows a graded system of punishment, with the punishment varying and the quantum of punishment being
dependent upon whether the offence pertains to small, commercial, and intermediate quantities of narcotic drugs and
psychotropic substances.
• For offences involving commercial quantities of drugs, a minimum penalty of ten years of rigorous imprisonment is
prescribed, which may extend to twenty years.
• Repeat offences attract one and a half times the penalty and, in a few cases, even the death penalty.
• By amendment to the Act in 1989, due to the serious nature of the offence, the sentence awarded under the NDPS Act
became non-commutable except for the sentence awarded for the consumption of drugs.
• Alongside these stringent provisions, the Act has procedural safeguards as follows:
• Personal search: Any person being searched has a right to be searched before a Gazetted Officer or a Magistrate
(Section 50).

For More Details Contact Us On: 0422-4954645; 9958604645


SA IAS ACADEMY UPSC (15/03/2024)
• Searches: Gazetted Officers of the empowered Departments can authorize searches. Such authorization has to be
based on information taken down in writing.
• Searches can be made under certain circumstances without a warrant (from a magistrate) or an authorization (from a
Gazetted Officer).
• Arrests: The person who is arrested should be informed, as soon as may be, the grounds of his arrest [Section 52(1) ] .
• If the arrest or seizure is based on a warrant issued by a magistrate, the person or the seized article should be
forwarded to that magistrate.
• The officer who arrests a person has to make a full report to his official superior within 48 hours.
Immunities:
• Officers: Officers acting in discharge of their duties in good faith under the Act are immune from suits, prosecution, and
other legal proceedings.
prosecution, and other legal proceedings.
• Addicts: Addicts charged with consumption of drugs or with offences involving small quantities will be immune from
prosecution if they volunteer forde-addiction. This immunity may be withdrawn if the addict does not undergo complete
treatment.
• Offenders: Central or state governments can tender immunity to an offender in order to obtain his evidence in the case.
This immunity is granted by the government and not by the court.
• Juvenile offenders: Juvenile offenders (below 18 years of age) will be governed by the Juvenile Justice (Care and
Protection of Children) Act, 2000.
• Immunities to diplomats as applicable.

87. With reference to Rhodamine-B, consider the following statements:


1. It is water-soluble chemical compound.
2. Its usage in food products is a punishable offence in India.
3. It is used in textile and paper industry.
How many of the statements given above are correct?
A Only one
B Only two
C All three
D None
Answer: C
Explanation:
Recently, the Tamil Nadu government banned the sale and production of cotton candy in the state after it was found that
the chemical Rhodamine-B was being used for making it.
• It is a water-soluble chemical compound.
• While it appears green in powder form, it turns vivid fluorescent pink when it comes in contact with water.
• It is a chemical commonly used for dyeing in the textile, paper, leather, and paints industry as a coloring agent that
helps in attaining the red and pink spectrum.
• "As per the Food Safety Standards Act, 2006, preparation, packaging, importing, selling and serving food items with
Rhodamine-B in wedding ceremonies and other public events is a punishable offence.
• It is an industrial dye which is not allowed in food anywhere in the world as it is toxic.
Why is it harmful?
• Researchers have identified that if food containing this chemical is consumed regularly, it can cause damage to the
cerebellum tissue in the brain and to the brainstem that connects the brain to the spinal cord.
• These damages can lead to functional abnormalities and can hinder humans' motor functioning.
• The chemical is toxic for humans and can cause oxidative stress on cells and tissues if ingested.
• It becomes particularly hazardous when it is mixed with food products, leading to cancer and tumour over time.

88. Consider the following statements regarding Quasar:

For More Details Contact Us On: 0422-4954645; 9958604645


SA IAS ACADEMY UPSC (15/03/2024)
1. It is an extremely active and luminous type of Active Galactic Nucleus (AGN).
2. All quasars are AGNs, but not all AGNs are quasars.
Which of the statements given above is/are correct?
A 1 only
B 2 only
C Both 1 and 2
D Neither 1 nor 2
Answer: C
Explanation:
An international team of astronomers said the recently discovered quasar named J0529-4351 was the brightest and
fastest-growing quasar discovered so far and had a supermassive black hole about 17 billion times the mass of the Sun.
About Quasar:
• A quasar is an extremely active and luminous type of active galactic nucleus (AGN).
• An AGN is nothing more than a supermassive black hole that is active and feeding at the center of a galaxy.
• All quasars are AGNs, but not all AGNs are quasars.
• Quasars are thought to form in regions of the universe where the large-scale density of matter is much higher than
average.
• They are among the most luminous, powerful, and vibrant objects known in the universe.
How are they formed:
• An active galaxy is one in which the central supermassive black hole is consuming large amounts of matter.
• The infall of matter into the black hole is so great that all the material can't enter the black hole at the same time, so it
forms a queue as a spiraling accretion disk.
• The matter-in the form of huge clouds-falls into the disk, with the inner parts of the cloud closer to the black hole
orbiting faster than the outer parts (just like planets closer to the sun orbit faster than those farther away).
• This creates a shear force that twists the clouds, causing them to bump into their neighbors as they move around the
black hole at velocities ranging from 10% of the speed of light up to over 80%.
• This friction from fast-moving gas clouds generates heat, and the disk becomes so hot-millions of degrees-that it shines
brightly.
• Some of the material in the disk is also funneled away from the black hole in a highly luminous, magnetically collimated
jet.
• The hot accretion disk and the jet combine to make the nucleus of the active galaxy shine so brightly that it can be seen
far across the universe.
• The brightest quasars can outshine all of the stars in the galaxies in which they reside, which makes them visible even at
distances of billions of light-years.
• Most quasars have been found billions of light-years away.

89. Which Middle Eastern country recently launched the satellites namely Mahda, Kayhan-2, and Hatef-1?
A. Iran
B. Iraq
C. Egypt
D. Jordan
Answer A
Explanation:
Iran launched three satellites into space using the Simorgh rocket: Mahda: A 32 kg satellite Kayhan-2: A nano-satellite
weighing less than 10 kg Hatef-1: A nano-satellite weighing less than 10 kg The satellites were launched from the Imam
Khomeini Spaceport in Iran's Semnan province.

90. Which state is home to the ‘Kutia Kondh Tribe’ classified as a Particularly Vulnerable Tribal Group?
A. Odisha

For More Details Contact Us On: 0422-4954645; 9958604645


SA IAS ACADEMY UPSC (15/03/2024)
B. Jharkhand
C. Andhra Pradesh
D. Chhattisgarh
Answer A
Explanation:
Two Kutia Kondh individuals, belonging to a Particularly Vulnerable Tribal Group in Odisha, have been invited as special
guests to the Republic Day parade in New Delhi. Each year, the Union Ministry of Tribal Affairs invites representatives
from such groups for Republic Day events.

91. The INDUS-X initiative, recently seen in the news, focus on advancing of:
A agriculture technology
B antipiracy measures
C high-tech cooperation in the defense sector
D renewable energy solutions
Answer : c
Explanation:
The defence ministry said the 'INDUS-X' summit in Delhi marks a "significant milestone" in the collaborative efforts
between the two countries.
About U.S.-India Defense Accelerator Ecosystem (INDUS-X):
• It was launched in June 2023 during the state visit of the Prime Minister of India to the US.
• Objective: To expand strategic technology partnerships and defense industrial cooperation between governments,
businesses, and academic institutions in India and the US.
• INDUS X will be a defence innovation bridge, which will inter-alia, include Joint Challenges, Joint Innovation Fund,
Academia engagement, Industry-startup connect, investment by private entities in defence projects, mentoring by
experts and niche technology projects etc.
It will focus on advancing high-tech cooperation and fostering joint research, development, and production opportunities
in the defence sector.
The initiative aims to explore possibilities for co-producing jet engines, long-range artillery, and infantry vehicles.
• India's Innovations for Defense Excellence (iDEX) and the Office of the Secretary of Defense (OSD), US, are leading
INDUS-X activities.

92. With reference to National Council for Transgender Persons, consider the following statements:
1. It is a statutory body.
2. It is headed by the Union Minister of Health and Family Welfare.
3. It aims to ensure to set up of transgender welfare boards in all states.
How many of the statements given above are correct?
A Only one
B Only two
C All three
D None
Answer : B
Explanation:
Recently, the National Council for Transgender Persons (NCTP) convened its pivotal meeting under the Chairmanship of
Secretary, Ministry of Social Justice and Empowerment.
• It is a statutory body constituted under the Transgender Persons (Protection of Rights) Act 2019 to safeguard the rights
of transgender persons in India.
• It is responsible for monitoring and evaluating the implementation of the provisions of the Transgender Persons
(Protection of Rights) Act, 2019 and making recommendations to the government on measures to promote and protect
the rights of transgender persons.

For More Details Contact Us On: 0422-4954645; 9958604645


SA IAS ACADEMY UPSC (15/03/2024)
Aims
• To focus on livelihood issues as well as to raise awareness about the trans community for achieving the goal of social
justice so that transpersons are accepted within families and in the larger society.
• To ensure to set up of transgender welfare boards in all states and also ensure that essential needs of the transgender
community, like housing, food, healthcare, and education are met.
Composition
The Union Minister of Social Justice & Empowerment is the Chairperson (ex-officio).
• The Union Minister of State for Social Justice & Empowerment is the Vice-Chairperson (ex-officio) of the council.
• The other members of the Council include representatives of various Ministries/Departments.
• Five representatives of the transgender community.
• Representatives of the National Human Rights Commission (NHRC) and the National Commission for Women (NCW),
representatives of State Governments and UTs, and experts representing NGOs.
Functions:
• It advises the Central Government on the formulation of policies, programmes, legislation and projects with respect to
transgender persons.
• It Monitors and evaluates the impact of policies and programmes designed for achieving equality and full participation
of transgender persons.
• It reviews and coordinates the activities of all the departments of Government and other Governmental and non-
Governmental Organisations which are dealing with matters relating to transgender persons.
• It redresses the grievances of transgender persons.
• Performs such other functions as may be prescribed by the Central Government.

93. Consider the following statements regarding Papua New Guinea:


1. It is located in the Southern Indian Ocean.
2. It is a constitutional monarchy and a member of the Commonwealth.
Which of the statements given above is/are correct?
A 1 only
B 2 only
C Both 1 and 2
D Neither 1 nor 2
Answer B
Explanation:
Dozens of men have been killed in a fresh outbreak of tribal violence in the remote highlands of Papua New Guinea
recently.
About Papua New Guinea:
• Location: It is an island country that lies in the south-western Pacific.
• It includes the eastern half of New Guinea (the world's second largest island) and many small offshore islands.
• Neighbours: Indonesia to the west, Australia to the south, and the Solomon Islands to the south-east.
• Capital: Port Moresby
• Terrain: It is mainly mountainous but has low-lying plains in southern New Guinea.
• The islands that constitute Papua New Guinea were settled over a period of 40,000 years by a mixture of peoples who
are generally referred to as Melanesians.
• Language:
• English is the main language of government and commerce. In most everyday contexts, the most widely spoken
language is Tok Pisin.
• Linguistically, it is the world's most diverse country, with more than 800 languages.
• Religion: The majority of Papua New Guinea's people are at least nominally Christian.
• Government:
• Papua New Guinea became self-governing on December 1, 1973, and achieved independence on September 16, 1975.

For More Details Contact Us On: 0422-4954645; 9958604645


SA IAS ACADEMY UPSC (15/03/2024)
• The country is a constitutional monarchy and a member of the Commonwealth.
• The British monarch, represented by a governor-general, is head of state, and the Prime Minister is head of
government.

94. Ratle Hydro Electric Project is built on which river?


A. Chenab River
B. Tawi River
C. Sutlej River
D. Kaveri River
Answer A
Explanation:
The government has announced the diversion of Chenab River water through tunnels to accelerate the 850-MW Ratle
Hydro Electric Project in Jammu & Kashmir. The project, located in the Kishtwar District, is a run-of-river hydroelectric
power initiative on the Chenab River. Developed by Ratle Hydroelectric Power Corporation (RHPCL) as a joint venture
between Jammu & Kashmir State Power Development Corporation and National Hydroelectric Power Corporation.

95. With reference to the Director General of Civil Aviation (DGCA), consider the following statements:
1. It is responsible for the regulation of air transport services in India.
2. It issues licenses and certificates to pilots, aircraft maintenance engineers and other aviation personnel.
3. It can investigate any incidents or accidents that occur within Indian airspace.
How many of the statements given above are correct?
A Only one
B Only two
C All three
D None
Answer C
Explanation:
The Directorate General of Civil Aviation (DGCA) recently issued a show cause notice to Air India in connection with the
incident where an 80-year-old man passed away after he walked to the terminal due to lack of wheelchair assistance at
the airport.
About Director General of Civil Aviation (DGCA):
• It is the regulatory body in the field of civil aviation primarily dealing with safety issues.
• It is an attached office of the Ministry of Civil Aviation.
• It is responsible for regulation of air transport services to/from/within India and for enforcement of civil air regulations,
air safety and airworthiness standards.
• It also coordinates all regulatory functions with the International Civil Aviation Organisation.
• Headquarters: New Delhi
• Functions and Responsibilities of the DGCA:
• One of the main functions of the DGCA is to ensure the safety of passengers and crew members on all flights operating
in India.
The organization conducts regular safety inspections of all airlines and aircraft to meet the required safety standards.
• It also investigates any incidents or accidents that occur within the Indian airspace and takes appropriate action to
prevent similar incidents from happening in the future.
• In addition to safety, the DGCA also plays a crucial role in the growth and development of the Indian aviation industry.
The organization works closely with airlines and airport operators to promote the industry's growth and improve the
overall travel experience for passengers.
• The DGCA also plays a crucial role in developing new airports and modernizing existing facilities to ensure that they can
meet the growing demands of the aviation industry.

For More Details Contact Us On: 0422-4954645; 9958604645


SA IAS ACADEMY UPSC (15/03/2024)
• The DGCA is also responsible for the regulation of air traffic in India. It works closely with the Airports Authority of India
(AAI) to ensure air traffic is managed safely and efficiently.
• The DGCA also plays a key role in developing new air traffic control systems and technologies to improve the overall
efficiency of Indian airspace.
• The DGCA is also responsible for issuing licenses and certificates to pilots, aircraft maintenance engineers, and other
aviation personnel.

96. The primary role of the Chinook/CH-47F helicopter is:


A Close Air Support
B Heavy troop and supply transport
C Reconnaissance
D Aerial refueling
Answer: B
Explanation:
A Chinook helicopter of the Indian Air Force made a precautionary landing following a "technical snag" near Punjab's
Barnala recently.
About Chinook Helicopter:
• Chinook/CH-47F is the U.S. Army's primary heavy troop and supply transport aircraft.
• It is an advanced multi-mission helicopter.
• It is manufactured by the American aerospace and defence firm Boeing for the US Army and international defence
forces.
• It is used for the transportation of troops, artillery, supplies, and equipment to the battlefield.
India has about 15 CH-47 Chinook helicopters.
Features:
• It is a twin-engine, tandem rotor heavy-lift helicopter.
• It contains a fully integrated digital cockpit management system.
• It has advanced cargo-handling capabilities that complement the aircraft's mission performance and handling
characteristics.
• It has a monolithic airframe with vibration reduction.
• Maximum Speed: 160 knots (296 kmph).
• It has a triple hook system, which provides stability for large external loads or the capacity for multiple external loads.
• It can carry up to 55 troops or approximately 10 tonnes of mixed cargo.

97. INS Sumitra’ is what kind of vessel?


A. Patrol vessel
B. Frigate
C. Destroyer
D. Aircraft Carrier
Answer:A
Explanation:
The Indian Navy’s INS Sumitra, a Saryu-class patrol vessel, rescued hijacked fishermen off Somalia’s east coast.
Commissioned in 2014, it’s the last vessel of its class, constructed by Goa Shipyard. Based in Chennai under the Eastern
Naval Command, its primary roles include EEZ surveillance, anti-piracy patrols, fleet support, maritime security, and
escort operations. The recent successful rescue underscores its vital contributions to regional security.

98. The Chars, recently seen in the news, are riverine areas mostly associated with:
A Brahmaputra River
B Godavari River
C Ganga River

For More Details Contact Us On: 0422-4954645; 9958604645


SA IAS ACADEMY UPSC (15/03/2024)
D Krishna River
Answer:A
Explanation:
Recently, the Chief Minister of Assam said his government would undertake a mission to reclaim the chars from
encroachers to safeguard the "ecological integrity" of the Brahmaputra Valley.
• The riverine areas (island) of the river Brahmaputra, locally known as "Char/ Chapori"
• The chars follow a peculiar pattern of migration.
• They are subjected to erosion on their upstream and deposition on the downstream, due to which they migrate
downstream.
This affects the geometry and location of the chars during floods almost every year.
Issues with Char area:
• The physical, sociological and institutional constraints on development of Char Areas are many and complex.
Such as virtual absence of all weather communication with the main banks, reliable means of intra and inter char
mobility, inadequate growth of transport and basic infrastructure like surface roads, health and drinking water, delivery
systems for farm inputs, marketing, agro-services electricity and education.
• The problems in the Char Areas are unique in nature, due to constant threat from flood and erosion during the rainy
season.
• The land mass is segregated and cut off from each other, making it very difficult for taking any major scheme for road
communication, irrigation, Power supply or setting of educational institutions.

99. With reference to Debts Recovery Tribunals, consider the following statements:
1. It was established under the Recovery of Debts and Bankruptcy Act 1993.
2. Its Presiding Officer is appointed by the Chief Justice of India.
3. The tenure of its Presiding Officer is 5 years.
How many of the statements given above are correct?
A Only one
B Only two
C All three
D None
Answer:B
Explanation:
Recently, the department of Financial Services secretary chaired conference of heads of Debt Recovery Appellate
Tribunals (DRATs) and Presiding Officers of Debt Recovery of Tribunals (DRTs) in New Delhi.
• The Debts Recovery Tribunals (DRTs) and Debts Recovery Appellate Tribunals (DRATs) were established under the
Recovery of Debts and Bankruptcy Act (RDB Act ) 1993.
• Objective: These are established to provide expeditious adjudication and recovery of debts due to Banks and Financial
Institutions.
• At present, 39 Debts Recovery Tribunals (DRTS) and 5 Debts Recovery Appellate Tribunals (DRATs) are functioning
across the country.
• Each DRT and DRAT are headed by a Presiding Officer and a Chairperson respectively.
• Under the section 22(2) in the Recovery Of Debts Due To Banks And Financial Institutions Act, 1993 DRT has the
following powers summoning and enforcing the attendance of any person and examining him on oath;
• requiring the discovery and production of documents;
• receiving evidence on affidavits;
• issuing commissions for the examination of witnesses or documents;
reviewing its decisions; dismissing an application for default or deciding it ex parte;
setting aside any order of dismissal of any application for default or any order passed by it ex parte;
any other matter which may be prescribed.
Composition

For More Details Contact Us On: 0422-4954645; 9958604645


SA IAS ACADEMY UPSC (15/03/2024)
• It is headed by the Presiding Officer to be appointed, by notification, by the Central Government.
• Qualification of Presiding officer: A person shall not be qualified for appointment as the Presiding Officer of a Tribunal
unless he is, or has been, or is qualified to be, a District Judge.
• Tenure: The Presiding Officer of a Tribunal shall hold office for a term of five years from the date on which he enters
upon his office and shall be eligible for reappointment.

100. Which naval ship was recently decommissioned after 38 years of service to the Indian Navy?
A. INS Suvarna
B. INS Nirupak
C. INS Kabra
D. INS Aditya
Answer: B
Explanation:
The Indian Navy bid farewell to INS Nirupak, a survey ship commissioned in 1985, after 38 years of service. Launched on
June 4, 1981, and based in Visakhapatnam, the vessel played a crucial role in hydrography, earning awards for excellence.
It actively participated in humanitarian operations, including the 2004 tsunami relief.

For More Details Contact Us On: 0422-4954645; 9958604645

You might also like